TOP カテ一覧 スレ一覧 100〜終まで 2ch元 削除依頼
●▲■語彙は力なり その22◆▲●
日本人が英語聞き取れない理由は?
(英和ではなく)和英翻訳者のための避難所
現役の翻訳者の集い(5)
新新TOEIC総合スレ
▲▼◆■英語学習の悩みを書いていくスレ■◆▼▲
紙の英和辞典について語るスレ 12
大坂なおみの英語
教師は2チャンやる暇あるなら単語でも覚えろ3
なんでも翻訳してやるから英文持って来い 21

なんでも翻訳してやるから英文持って来い 18


1 :2019/01/27 〜 最終レス :2019/05/28
前スレ
なんでも翻訳してやるから英文持って来い 17
https://lavender.2ch.sc/test/read.cgi/english/1537611519/

2 :
乙です

翻訳の反対というか、
こう伝えたい時にはどう書けば?っていうスレ、ありましたっけ?

3 :
Our colleagues feedback you send the goods to us more and early than we request ,
could you help check if can follow our request date shipment ?
We need to control stock .
経緯としては、出荷日を伸ばしてほしいと客先から連絡がありましたが、それは出来ない、出荷を早くさせてくれと伝えていました。
その後に、このような回答が来ました。
なんと伝えてきているのでしょうか。
宜しくお願い致します。

4 :
Our colleagues feedback    send the goods to us more and early than we request ,
could you help check if  you   can follow our request date shipment ?
お知らせしたとおりですので、しばらくお待ち願えませんか。
We need to control stock .

5 :
同僚から聞いたけどよー、おめーまた商品送って来たんだってな!しかもこっちが頼んだ期日よりも早く。
あのさあ…ちゃんとこっちが頼んだ出荷日を守ってくんないかなあ?
在庫を調整しなきゃなんないわけよ、わかる?
(実際はこんな口調ではないが込められた憤りはだいたいこんなものと想像)

6 :
>>4
>>5
大変助かりました!!

7 :
His left arm hangs limply at his side as he mounts his horse.
よろしくお願いします

8 :
彼は馬にまたがったら左腕をだらりと垂らした

9 :
海外の通販で購入したものが届かずに追跡番号も通知されないままなので相手にメールを送ったのですが数日経っても返信されないので、サイトの公式フォーラムで「あなたにメールしたので返信してね」的なメッセージを投稿したのですが、このようなメッセージが帰ってきました。
Hello, sorry about that, message forwarded.
これはどういう解釈をしたらいいですか?
自分が出したメールが届いていないのか、それとも相手が私にメールを出したって意味で書かれているのか・・・
どちらにしても、こっちに相手からのメールは来てないんですがねw

10 :
>>9
メッセージ(メール)を(係の人に)転送しといたよ

11 :
>>10
ありがとうございます

12 :
すいません、よろしくお願いいたします m(_ _ )m

Before concluding these lines of little interest to you, in view of the proximity of the turn of the year, I congratulate you and your highly esteemed dependents on the coming year of 2020.

一応自前で意訳したのはこれですが、
「これらのちょっとおもしろい線を決める前に、もうすぐ年の変わり目ですから、私はあなたとあなたがとても尊敬されている旦那さんが来年1902年、最高の幸運を願って祝福します。」
どうしても「Before concluding these lines of little interest to you,」がこなれた日本語にできませんものでして… m(_ _ )m

13 :
>>12
自前の訳文は「2020」と「1902」まちがえてました orz

14 :
>>13
あなたにとっては興味がないこれらの文章を終える前に=つまらないことをつらつらと書いてきましたが最後に

15 :
>>14
あ、なるほど! littleがすっぽり抜けてました
ありがとうございました m(_ _;)m

16 :
おはようございます。
外国人がまた行きたいの意味で
I went to go back.
と言っていたんですが文法的に合ってます?
なんでそういう意味になるか分かりません。

17 :
youはあなたなの?
Before concluding these lines of little interest to you
これらのlineについてはさておいて
で意味通るの?

18 :
>>16
I want to go back なら合ってる。

19 :
洋書の文章になります。
It can be daunting for a family to discuss bad news.
Sometimes, if the bad news is broken only to the patient, or only to a family member, that individual can find themself with the burden of knowing a truth they dare not speak.
This can lead to a whole conspiracy of silence that isolates people from each other at the very time they need to draw upon each other's strength and support.
It is possible to be lonely despite being surrounded by a loving family, as each person guards their secret knowledge for the love and protection of another.

まず、whole conspiracy of silenceの正しい訳し方を教えていただけると幸いです。
conspiracyについては、わたしは「全体の沈黙の作為」と訳してみたのですが… 「陰謀」は固すぎる気がします。 文脈的に「わざと黙っている」のようなニュアンスなのかなとは感じているのですが
もう一つ、4文目のasの用法についてもお願いいたします。
こちらは理由のasという認識で構わないのでしょうか?
「それぞれがお互いを愛し、そして守るためにその秘密の知っていることを守るため、愛する家族に囲まれていたとしても(以下略)」のように訳してみたのですが、間違いがあれば是非とも教えていただきたいです。

以上2点になります。よろしくお願い致しますm(_ _)m

20 :
>>19
A whole conspiracy of silenceは家族全員が悪い知らせを伝えないという手段を取った場合の無意識的な帰結を指すから、「各人が口裏を合わせたかのように沈黙してしまう」くらいの意味では?
asは理由でいいと思う

conspiracy of silence
沈黙の共謀◆公開すると不利な立場に追い込まれる事実に対して、沈黙を守ろうという申し合わせ。通例恥ずべき行為とされる。

21 :
家族を思いやって言葉にしないということでしょ。
仲の良い家族と一緒にいても秘密を守るのでlonelyになることはありうる。

22 :
>>18
あー、字幕が間違ってたんですかね
ありがとうございます

23 :
>>21
いつも後出し

24 :
Karen,Akari and the red hair of Minori are very beautiful

25 :
>>20 >>21
返信が遅れ申し訳ないです
ありがとうございます

26 :
よろしくお願いします

one of those old days

27 :
昔懐かしい日々の内の1日。

28 :
if you’re not at least suggesting to meet up half way, you’re a POS

出会い系のプロフィール文です
お願いします

29 :
POS=パブリックオRシステム

いつでもやらせてくれるということ

30 :
Japanese empire was destroyed by a slip of paper cover with obscure symbols

Red Planet Mars と言う映画の中のセリフの1部分です

31 :
>>29
パブリックオチンコシステムの可能性はないでしょうか

32 :
>>30
Japanese empire was destroyed by a slip of paper covered with obscure symbols.
大日本帝国は暗号が記された紙一枚のせいで破壊された。(暗号を解読されて)

33 :
32さん
ありがとうございます。
おかげさまで前後の文章の流れがよく理解できました。

34 :
Another Trump insider appears bound for the old (Roger)stone hotel.
How long before Trump supporters realize that you don’t surround yourself
with dirty guys unless you’re dirty yourself?

翻訳お願いします

35 :
>>34
また一人、トランプ陣営の内部者が、例の石造りのホテル(刑務所)行きになりそうだ。
トランプ支持者たちときたら、一体いつになったら、よからぬ者たちに取り巻かれるような人間は、その本人がよからぬ者だからこそだということに思い至るのかね。

36 :
>>35
ありがとうございます

37 :
Yet if there's no reason to live without a child, how could there be with one?
To answer one life with a successive life is simply to transfer the onus of purpose to the next generation; the displacements amounts to a cowardly and potentially infinite delay.
Your children's answer, presumably, will be to procreate as well, and in doing so to distract themselves, to foist their own aimlessness onto their offspring.

一時間以上悩んでわかりませんでした。質問させてください。

38 :
しかし、子無しの人生に意味がないならば、子持ちの人生には一体どんな意味があるというのか。
子を持てばもう一つの人生を歩めるのだという回答は、望みを果たす責任を次の世代に転嫁しているにすぎない

39 :
そうした責任転嫁の結果、情けないことだが、望みの達成はいつまでも先延ばしされることになるかもしれない。
おそらくその子供が出す答えも、親と同じく、子を産むことだ。
そうして現実から目をそらし、自らの漫然とした生き方を子孫に押し付けるのだ。

40 :
しゅうせい

そうした責任転嫁の結果、情けないことだが、先延ばしされつづけて望みが叶わない可能性もある。
おそらくその子供が出す答えも、親と同じく、子を産むことだ。
そうして現実から目をそらし、自らの漫然たる生き方を子孫に押し付けるのだ。

41 :
>>37
どうでもいいことかもしれないが、その引用文は Goodreads のサイトからのものだろうけど、
typo があるな。

>>the ★displacements★ amounts to a cowardly and potentially infinite delay.
(Goodreads のサイトより)
https://www.goodreads.com/quotes/355494-yet-if-there-s-no-reason-to-live-without-a-child

この displacementS は間違いだが、Google Books 上にある同じ本では、この下のリンク先にあるように、
ちゃんと displacement と単数になっている。


https://books.google.co.jp/books?id=GNp0SptFYg8C&pg=PA302&dq=%22displacement+amounts+to+a+cowardly%22&hl=en&
sa=X&ved=0ahUKEwjqw7zrh6jgAhXGabwKHbvkCVIQ6AEIKjAA#v=onepage&q=%22displacement%20amounts%20to%20a%20cowardly%22&f=false


"We Need to Talk About Kevin" by Lionel Shriver

42 :
>>37
ところで、この "We Need to Talk about Kevin" という小説と映画は、
面白そうだな。映画の trailer を見て気に入ったので、さっそく DVD を
注文した。小説もいずれは読んでみるよ。紹介してくれた質問者に感謝。

43 :
>>39 a successive life というのは生んだ子供の事だったのですね…! ありがとうございます本当に助かりました

44 :
>>42
インドのanti-natalisnが親を訴えたってニュースからここまでたどり着きました。
プライムで配信されてたので僕も映画見ました。お勧めてですよ〜。

45 :
Would you rather travel or be in any hotel you wanted for free?

お願いします。

46 :
>>45
(この1文だけで解釈するなら)
あなたは旅(移動)するのと、どこでも好きなホテルに無料でいるのと、どちらがいいですか。

47 :
>>46
どう言う状況なんだろうね?高いホテルと比べてるのかな。

48 :
このスレ全体で言えることなんだけど、前後の文章ものせないと意味変わる場合あるよね

49 :
the EU isn't strong enough to be weak right now
お願いします

50 :
いまEUはやばい

51 :
>>49
今現在EUには(脱退するイギリスに譲歩できるほどの)余裕が無い。

52 :
Trump's speech was light on new initiatives to further stoke growth in an economy seen as losing momentum as it began 2019.

よくわかりません…、お願いします

53 :
トランプのスピーチは経済成長を掻き立てる独創性に欠けてて
2019年始めから勢いを失っていっているように思える

54 :
>>53
ありがとうございます

55 :
You can answer one question. In the news they said you were the famous [ニックネーム].
The one they made a film about. Is that true?

昔の有名な犯罪者と同じ名前・やり口の人物に対して質問しているシーンです。
filmのところがよくわかりません。よろしくお願いします。

56 :
>>55
ひとつだけ質問に答えてほしい。ニュースで、あんたがあの有名な〇〇だと言ってた。
あの、映画にもなったという…。本当なのかい?

the famous xxとthe one (who they made a film about)は同格(言いかえ)

57 :
>>56
ありがとうございます!

58 :
宜しくお願いします。

Finally, proponents claim it helps protect liberalism at home
by eliminating authoritarian states that otherwise might aid the illiberal forces
that are constantly present inside the liberal state.

59 :
Queen - Live at LIVE AID 1985/07/13
クイーンの動画へのコメントになります。

Well, I feel bad for whoever had to go on next.?

60 :
>>59
この次に演奏しなければいけなかったグループは気の毒だな

61 :
>>59
すいません。解決しました。

62 :
>>60
ありがとうございます。

63 :
“the child is feeding a dog”

これって子供が犬に餌を与えていると訳すべきなのか子供が犬を飼っていると訳すべきなのかどっちですか?

64 :
餌を与えてるのほうが良いかもしれない。
定冠詞のtheに注意。

65 :
>>58
Finally, proponents claim it helps protect liberalism at home
by eliminating authoritarian states that otherwise might aid the illiberal forces
that are constantly present inside the liberal state.
最後に、推進派はこのようにも主張している。これ(it)は国内のリベラリズムを保護するのに役立つと。リベラルな国のうちにも常に存在している反リベラル勢力を、助長していた可能性もある専制国を排除することになるので。

66 :
I fought so that no child born into slaver's bay would ever know what it meant to be bought or sold.

これはso that構文なのでしょうか?
what it is meant ではなくwhat it meant なのもよくわかりません
自分の英語レベルが低くてすみません
翻訳お願いいたします

67 :
>>66
私は、「買われる」とか「売られる」という言葉の意味を知らないまま奴隷商人湾で生まれる子供をなくすために戦う

68 :
「何時ならあいてる?」
「19時頃かな」の返信としての
「I think later..」はどんな意味でしょうか

69 :
>>66
I fought so that no child born into Slaver's Bay would ever know what it meant to be bought or sold.

どうやらこれは、Slaver's Bay という名前のゲームに出てくる台詞のようだな。

(1) I fought --- 私は闘った。
(2) so that S would V --- S が V するように(SがVするように仕向けるために)
(3) no child born into Slaver's Bay --- Slaver's Bay (奴隷商人の湾) に生まれ落ちた子供は、誰一人として〜しない
(4) no child ... would ever know ... --- (これこれこういう)子供は、誰一人として〜を知ることが決してない
(5) what it meant to be bought or sold
   --- 買われたり売られたりするということがどういうことを意味しているか(実際にはこれは過去形)ということ
(6) so that no child ... would ever know what it meant to be bought or sold
   --- (〜する)子供が誰一人として売買されるということがどういうことなのかを決して知ることのないように
(7) I fought so that no child born into Slaver's Bay would ever know what it meant to be bought or sold.
   --- 奴隷商人の湾に生まれ落ちた子供が、誰一人として売買されるとということが
   どういうことなのかを決して知ることのないよう、私は闘った。

(8) 少しだけ意訳
奴隷商人の湾に生まれ落ちた子供が、誰一人として売買されるなどと言うことを絶対に経験しなくて済むよう、私は闘ったのだ。

(9) what it meant to be bought or sold
これを現在形にすると、
(10) what it means to be bought or sold (自分が売買されるということがどういうことを意味するかということ)

(11) what を a tragedy に変えてみると
It means a tragedy to be bought or sold.
そしてこの it は to be bought or sold を意味するので、(11) は次の文と同じ意味。

(12) To be bought or sold means a tragedy.
(自分が売買されるということは、悲劇を意味する)

70 :
>>66
私は、スレイバーズベイに産まれた子供の誰も(人身)売買されるのがどういうことなのかを知ることがけっしてないようにと闘った。

目的を表すso that
what it meant の it は仮主語でto be 〜を受ける
スレイバーズベイは固有名詞みたいなので訳さなかったけどslaverは奴隷商

71 :
Does anybody have experience with renting an apartment for a single person for a month in SEA. preferably in Chiang Mai, Bangkok or Vietnam.

アメリカ人がアジアで短期のアパートを探している文ですがSEAとはなんですか?

72 :
おそらく東南アジア  
Chiang Mai,チェンマイ  Bangkok or Vietnam.

73 :
>>72
ありがとうございます。

74 :
>>67
>>69
>>71
詳しい解説ありがとうございます!
理解できました!

75 :
We are aware that we have many advantages, and we value showing appreciation for this by sharing with others who may have less.

よろしくお願いします。

76 :
>>75
私達は自分たちが多くの点で有利な立場にあることを承知しており、自分たちほど有利な立場にはない可能性のある他の人たちと分け合うことによって、この事実への感謝を明らかにすることを貴ぶものである。

77 :
>>76
ありがとうございます
しかし日本語難しい

78 :
今恵まれてる環境にあっても感謝の気持ちを持ち弱者に分け与えろ。

79 :
As an Academy voter, I'm not happy about
the decision to squeeze down some of the awards.

翻訳お願いします

80 :
アカデミー賞有権者としてはある部門の賞が縮小されるのは不満である。

81 :
>>79
アカデミー会員として、一部の賞をすみっこに追いやるという決定には自分は満足していない。
(アカデミー賞のうち4部門の表彰をCM中に済ませてしまうという決定)

82 :
So passeth, in the passing of a day,
Of mortal life the leaf, the bud, the flower,
Ne more doth flourish after first decay,
That erst was sought to deck both bed and bower,
Of many a lady, and many a paramour;

エドマンド・スペンサーのThe Song of the Roseの一節です(『イギリス名詩選』より)
できれば直訳をお願いします

特に分からないのは、
・一行目のpassethって何?
・2行目のOfがどこにかかってるか(a day?)
・2行目なんで"mortal life"と"the leaf"の間にコンマがないのか(A, B, C and Dとなるはず)
・3行目”Ne more”の意味(No more?)

83 :
>>82
(1) So passeth, in the passing of a day,
(2) Of mortal life the leaf, the bud, the flower,
(3) No more doth flourish after first decay,
(4) That erst was sought to deck both bed and bower,
(5) Of many a lady, and many a paramour;


(3) の Ne は、No が正しいはず。

上の (1) から (5) を、現代語でしかもごく普通の語順で書き直してみる。

As a day of mortal life passes,
the leaf, the bud, and the flower thus pass and no more flourish after first decay,
which was at first sought (in order) to decorate both the beds and bowers
of many ladies and many lovers.

俺なりの和訳:
人生の一日が過ぎ去る中で、
木の葉、つぼみ、そして花がこのようにして散っていき、
最初に衰えたあとは、もう栄えることはない。
この最初の衰えは、かつてはたくさんの貴婦人やその愛人たちの寝床と寝室との
両方を飾るために求められたのだ。

(このように訳してみたが、ちょっとわけのわからない部分がある。しかし
英語原文を俺なりに素直に解釈したら、このようにしかならない。)

(続く)

84 :
>>82 への回答の続き

passeth は、passes のこと。pass という動詞の三人称単数現在。
in the passing of a day of mortal life が一塊で、「人生の一日が過ぎ去る中で」。
so passeth とは so passes という意味だが、「このようにして過ぎ去るのだ」という意味。

その主語は、the leaf, the bud, (and) the flower だ。その主語が複数なのに
なぜ so passeth (= so passes) と三人称単数になっているかというと、
昔(少なくとも400年くらい前までのイギリス)では、主語が複数であっても、そのあとの
動詞を単数で受けることがよくあったことと関係していると思う。少なくとも Shakespeare では
主語が複数なのにそのあとの動詞が単数ということはよくある。

first decay, のあとにある that とあるのは、その that が関係代名詞であり、その前の
both bed and blower を先行詞としているはず。

many a lady は many ladies のことで、many a paramour は many paramours という意味。

(3) の doth は does のこと。その主語は (2) の the leaf, the bud, and the flower だ。

(5) の of many a lady and many a paramour を受けているのが (4) の bed and bower だが、
現代語(しかも散文)であれば、the beds and bowers となろう。

85 :
>>83-84
ありがとうございます

(4) That erst was sought to deck both bed and bower,
の関係代名詞 that の先行詞は”the leaf, the bud, (and) the flower”ではないですか?

つまり:
The leaf, the bud and the flower <that at first was sought to deck both bed and bower> pass and
no more flourish after first decay,
(かつては寝床や寝室を飾るために求められた―つまりかつては美しかった―木の葉、つぼみ、花もまた、このように散ってゆき、
最初に衰えたあとは、もう栄えることはない。)

そうだとすると、ここでも先行詞が複数なのに関係代名詞節の動詞が単数形(was)になってしまいますが、
これも主語-動詞間と同様、昔の文ではよくあることなんでしょうか?

86 :
>>83
素晴らしい訳ですね。感動しました。

87 :
what food places/restaurants do you recommend? I wanted to try Five Guys, Taco Bell, Joe's Pizza

『honest to god NPC tier behavior, just stay home and save your goyim coupons』

『』の部分がわかりません。お願いします。

88 :
>>85
あなたの言う通り、that の先行詞は確かに the leaf, the bud, (and) the flower
ですね。僕が一人で読んでいた時は、この点がどうしてもよくわからなくて
辻褄が合わなかったのでした。ありがとうございます。

それから、ここでの was もやはり、現代語であれば were になるところを、
400年以上前の Early English においては単数になることもよくあったそうです。
これについては、

(1) G.A. Abbott, "A Shakespearean Grammar," 332番と334番

という本の中に、Early English においては

(1a) Northern English においては they hopes となり
(1b) Midland Engish においては They hopen となり
(1c) Southern English においては they hopeth となった。

と書いており、さらに Shakespeare の中では韻律をそろえるためもあって、
(3b) の形を頻繁に使った、と書いています。具体的に言えば、有名な
"Romeo and Juliet" の冒頭付近で、次のような一節が出てきます。

(続く)

89 :
>>85 への回答の続き

Whose misadventured piteous overthrows
★Doth★ with their death bury their parents' strife.
https://owlcation.com/humanities/Romeo-and-Juliet-Prologue-Analysis-Line-by-Line

この部分を現代語に(しかも僕なりにわかりやすい英語に)書き直すと、次のようになります。

Whose unlucky miserable falls
bury their parents' conflict with their deaths.
(ロミオとジュリエットというこの若い二人の惨めな転落が、
自らの死によって親たちのあいだの紛争を終焉させたのである。)

ここで doth とは does のことであり、単数を受ける動詞の形です。
しかしその主語はその前の overthrows なので、複数なのです。これについて、

The Arden Shakespeare, Third Series, "Romeo and Juliet," edited by Rene Weis, p124

にも、次のように書いています。つまり、やはりここでの Shakespeare の一節でも、この当時の
イングランド南部英語によくあった三人称複数主語に対する単数みたいな形の動詞を
使っていると書いています。

90 :
>>87
>>what food places/restaurants do you recommend? I wanted to try Five Guys, Taco Bell, Joe's Pizza
>>『honest to god NPC tier behavior, just stay home and save your goyim coupons』

(1) goyim については、
https://www.urbandictionary.com/define.php?term=goyim
このサイトを見るとおそらく、「ユダヤ人以外の国すべて」を指すように思える。
だから goyim coupons は「異教徒(ユダヤ人以外の連中)が行くレストランのクーポン」
という意味だろう。

(2) NPC tier behavior については、
https://www.urbandictionary.com/define.php?term=NPC
このサイトでの解説を読むと NPCは簡単に言えば「馬鹿」のことだろう。
そうすると NPC tier とは「馬鹿の階層の連中」ということで、NPC tier behavior
とは「馬鹿どもの振る舞い」ということだろう。

(3) そうすると
honest to god NPC tier behavior, just stay home and save your goyim coupons
これの全体の意味は、だいたい次のような感じだろう。

俺なりの意訳:
「はっきり言えば、それは自分でものが考えられない馬鹿の振る舞いだよな。
外出なんてしないで、異教徒(ユダヤ教徒以外)が行くレストランのクーポンが
せっかくあるんなら、使わないで置いとけよ。」

91 :
>>88-89
ありがとうございます(とても勉強になりました)

92 :
>>90
ありがとうございます。

93 :
I saw ted when i was elementary school student with my mother. So I really Awkward.

94 :
I was really awkward.でした。すいません

95 :
I bet you wouldn't black crystal out in a 3v1, but your vagina starts sweatin in a 1v1 you gotta run

Join the sunbros you twat

どういう意味ですか?お願いします

96 :
>>93
映画『TED』は小学生のときにカーチャンと見に行ったよ。だからめっちゃ気まずかったw

97 :
it’s my private belief that he would have broken his word if the indignation of one irrepressible young lady had not found vent in a hiss.
まったく分かりません。よろしくお願いします。

98 :
>>97
私の考えでは、もしこのときある一人の我慢のきかない若い女性が、憤慨のあまり歯の隙間からスーッと音を立ててさえいなかったら、彼は前言を翻していたことと思う。

99 :
>>98
ありがとうございます!
よく分かりました。

100 :
○○ feat.○○ Fairlife

どんな意味合いで受けとれば良いのか分かりません。
」feat.」「Fairlife」単体の意味合いでも構いません。
よろしくお願いいたします。

101 :
>>100
Fairlifeという音楽ユニットの曲名じゃね
ふつう、feat.誰それはアーティスト名のほうにくっつけてゲストなど特別扱いの人を明示するものだけど
このユニットは曲名のほうにつけるのを流儀にしてるみたいだ

102 :
All that is here will be sunlight.

2つの解釈がありえるけど、どっちがいいんだろう…?

(1) ここにあるものはすべて太陽の光に変化するだろう
(2) ここはすべて太陽の光で満たされるだろう

問題となるのは"All that is here"の解釈

今ここにある具体的なもの(例えばPCなり机なり)が、未来において太陽の光に変化するのか((1)の場合)
それとも、
抽象的なものとしての”ここにあるもののすべて”が未来において太陽の光だけになる、
つまり今ある机やPCはそのままどこかへ消えて太陽の光がどこからかやってくるのか((2)の場合)

103 :
そうなの?
ぱっと見、ここに残るのは日光だけ みたいな文なのかと思った。英語勉強しないとダメだなぁと思いました。

104 :
>>101レス有難うございます。
ユニット名なのは知ってますが、
feat.(feature)の考えられる意味合いと

Fairlife(Fairな生活?) 日本語にした場合の考えられるニュアンス? どう日本語にすれば良いか悩んでます。

105 :
>>80
>>81

ありがとうございます

106 :
>>103

例えば、
All that is a member of Grade-3: Class-A will be a doctor (3年A組の奴らはすべて医者になるだろう)と言った場合、
(3年A組はエリートの集まりで)具体的な3年A組のメンバーすべてが医者に変化するという意味
一方、
All that is a member of Grade-3: Class-A will be a boy (3年A組はすべて男になるだろう)」といった場合、
今の具体的な3年A組のクラスメイトが男に変化するのではなく、未来において(男子校化されるなどして)3年A組は男だけで構成されるだろうという意味

…になるかなぁと思って、"All that is here"はどっちだろうと思ったのですが(そもそもが間違っている?)

107 :
全てのものが太陽の光になるってのは英語というより、文章の意味としてどういう意味?
(2)は103と同じ意味だろうね。要は実質的には何も残ってないという意味かもしれないけど。誰か答え教えて。

108 :
文脈次第

109 :
*
文脈は、これだろ?

★All that is here will be sunlight★
Empty the name that you sign
Of the given I means mine
Like the shape is a shell
And the inside is whole
You behold, you behold

これは歌詞だけど、これについては英語ネイティブらしき人たちも
https://www.reddit.com/r/John_Frusciante/comments/4l5vy5/lyrics_discussion_shelf/
このリンク先でいろいろ論議している。

110 :
>>107
色即是空。
この世の万物は形をもつが、その形は仮のもので、本質は空(くう)であり、不変のものではないという意。
あるのは太陽の光のみ。

111 :
その手の概念でも will 使うの?

112 :
日本語にない未来形を吟味しよう。

113 :
>>112
授業で普遍の真理やことわざなどは現在形で表すと習ったもので.....
出来れば他の例も学びたいので教えていただければありがたいです。

114 :
The lesson contained in Russell's Paradox and other similar examples is
that by merely defining a set we do not prove its existence.

この英文の意味は以下のような意味だと思いますが、この「by」の意味は何ですか?

ラッセルのパラドックスや他の似た例に含まれる教訓は、ただ単に集合を定義するだけで、その存在を証明しないということである。

115 :
>>114

ちなみに、集合論の本からの引用です。

116 :
前置詞で「何々によって」の意味じゃない?
副詞句?が前にでた形じゃない?

117 :
>>114
proveにかかり、その手段
「ある集合を定義しただけではそれが実在すると証明したことにはならないということ」

118 :
>>116
>>117

ありがとうございました。

119 :
以下の英文の詳しい分法的な構造を教えてください。
特に「but」の意味は何でしょうか?

意味は大体こんな感じだと思います:

我々は、中でも、自然数、有理数、実数、関数、順序などの基本的な性質を含む、
知られている限り無矛盾である、集合についての多数の真実まで行き着くことになる。


We end up with a body of truths about sets which includes, among other things,
the basic properties of natural, rational, and real numbers, functions, orderings, etc.,
but as far as is known, no contradictions.

120 :
>>119
We end up with a body of truths about sets
- sets which includes the basic properties of 〜, but (includes) no contradictions.
butは接続詞で、意味は逆説。「Aを含むがBを含んでいない」
で意味は通るのではないかな(数学で使う表現に関して自信がない)
as far as is knownはそういう決まり文句ということですませて、くわしい説明は勘弁願いたい。文の流れ的に「これまでに確認されているかぎり」といった気持ちかと

among other thingsは「他にも要素はありますが」という気持ちが含まれているとは思いますが「中でも」でもまったく誤訳ではない

121 :
>>119
あえて原文を前から順番に、英文の流れの順序のままに
「意訳」していくと、こんな感じになろうか。


(1) We end up with a body of truths about sets
われわれは最終的に、集合についていくつかの真理に行き着くことになるのだが、

(2) which includes, among other things,
その中には、他にいろいろある中で、

(3) the basic properties of natural, rational, and real numbers, functions, orderings, etc.,
自然数・有理数・実数・関数・順序などの基本的な性質が含まれているが、

(4) but as far as is known, no contradictions.
知られている限りでは、矛盾は含まれていない。

★なお、restrictive relative pronoun (限定用法) と
non-restrictive relative pronoun (非限定用法) との違いについては、
きちんと理解したうえで、それでもなおかつあえて上のように「意訳」しておいた。

英文を読むときには、こんなふうに考えながら読むのが普通なのだろうと感じている。
ただ、日本語と英語とは考え方の順序というか流れが逆である場合が多いので、漢文の書き下しみたいに
下から上へと戻らないと仕方がないこともよくある。ただ、できれば上のように解釈した方が
いいよね。

122 :
>>120
>>121

ありがとうございました。

123 :
数学とか物理なんかはまさに
>>121
こういう読み方をした方がいいと思うぞ
特にこの手の学問やって書いてる奴らは理屈っぽいから、その使い方もワンパターンっていうと失礼だが、
あとから付け足す言い方が基本なのだ

124 :
>>119
includeの目的語はこうかな
・the basic properties of natural, rational, and real numbers,
・functions,
・orderings,
・etc.,
・no contradictions.

which includesから先行詞はa body(>>120は間違い)

125 :
これらの翻訳をお願いします

Pokud ano, kdo jedn?
Myslm, e bych to mla dlat

Mte pravdu.Pouil jsem peklada!
Chtli jste jen vdt, o em jsem mluvil, e?

126 :
>>125
キリル文字(ロシア語の文字)にそのまま直せば、何となくロシア語っぽくなる
みたいな言語だな。俺の知っているわずかなロシア語に少し似ている。
おそらくは、ロシア語またはその方言、あるいはそれに似たスラブ諸語のうちの
いずれか(たとえばブルガリア語・スロベニア語・チェコ語など)だろう。

127 :
>>125 がブルガリア語だと想定して Google Translate にかけたら、
次のような結果。


If so, who does it?
I think I should do it

You are right.
You just want to see me talking about them, eh?

128 :
>>125 がチェコ語だと想定すると、


If so, who does it?
I think I should do it

You are right.
You just want to see me talking about them, eh?

129 :
>>125 がポーランド語だと想定してみると、
結果は惨憺たるものだから、おそらくは
>>125 はポーランド語ではないと思う。
チェコ語やブルガリア語だと想定したら、
まあまあの結果が出ているから、おそらく
それに近い言語なのだろう。

130 :
>>125
これはロシア語の文字(キリル文字)に転換すれば
ロシア語にいくらか近くはなるけど、ロシア語ではない
と思う。ロシア語なら、俺でもほんの少しはわかるからな。
しかしこれはロシア語ではないように思う。

131 :
>>123

ありがとうございます。

>>124

ありがとうございます。

include の目的語を以下のように分類しなかったのはなぜですか?

・the basic properties of natural, rational, and real numbers, functions, orderings, etc.,
・no contradictions.

132 :
>>124

「a body」が先行詞だから、「includes 」、「is known」となっているんですね。

133 :
>>131
並列させるときは最後の要素の前にand(やor等)を付けるというルールから。
butが同じ役割を果たしている


>>132
isのほうは無関係だね
本文の主語の単複とは無関係につねにisだから

134 :
>>131
俺はetc.までの数の例とno contradictionsは同格でならべてるだけだと思う
but as far as is knownと注意をつけて、注釈を追加するために

ラストにandで終わるルールは、日本語にはない、ラストを明確にさせる理由からだけど、
後ろから最後に説明つけたら最後もわかるし

135 :
>>133
>>134

ありがとうございました。

136 :
翻訳お願いしますm(_ _;)m

before i get the account, thats when i would ask you to let staff hold the funds, so you cant like decide not to buy it (not saying you would but stuff happens)

137 :
>>136
文脈がわからないし混乱してる書きぶりなので自信はないけど…

私がそのアカウントを手に入れる前に、あなたにお願いしておきたいことなんだが、
その資金はスタッフに預からせておいて、たとえばそれを購入しないという決断をあなたができないようにしてほしいんです(あなたがそうするだろうというわけではないですがそういうことはままあるので)

なんだこれ、詐欺か?

138 :
likeの場所と用法も、わけわからんかったが、ネイティブの英語じゃねえってことか
最後の()内前半わからんかったし
なるほど、詐欺かー

139 :
Four-year-old Kun is an only child — this is,
until his parents bring home a baby sister named Mirai
(the Japanese word for future)
and the boy gets rattled by the new addition to the household.
Not much there for a full-length feature film … or so you’d think.
Except you are in the presence of
Japanese animation artist Mamoru Hosoda (Wolf Children, Summer Wars),
who again transforms the seemingly conventional
into a magic carpet ride of time and memory.
It makes all the difference.
Released in an English-dubbed version,
Mirai emphasizes that family is at the center of the universe for the lad,
voiced here by Jaden Waldman.
Hosoda, who founded the production house Studio Chizu,
pays particular attention to the living circumstances
in the modernist home designed by Kun’s architect father (John Cho),
including a backyard with just one tree.
This totem of nature is symbolic of the days
when the older kid alone dominated this world,
with support from dad, mom (Rebecca Hall),
a kindly grandma (Eileen T’Kaye) and the family’s lovable dog, Yukko.
Each detail of the house is subtly rendered to show the impact
on a child whose life has been knocked out of joint by a bawling,
baby (Victoria Grace).
When his mother goes back to work,
leaving Pops to mind the kids, the tantrum-throwing Kun
—he’s not above popping his sister on the nose or whacking her with a toy train
— feels lost enough to run away from home.

未来のミライ のレビュー
翻訳お願いします

140 :
>>139
4歳のくんちゃんは一人っ子だ――そうこれは、彼の両親が赤ちゃんである妹のミライちゃん(日本語で未来を表す言葉)を家に連れてきて、くんちゃんがこの家族の新たな一員に振り回されるようになるまでは。
長編のストーリー映画にしては中身が薄いような……とあなたは思うかもしれない。
ただしそれも、日本人アニメアーティストの細田守氏(「おおかみこどもの雨と雪」「サマーウォーズ」)が関わっているとなれば話は別で、氏は再び、見たところありきたりな題材を、時と記憶に関する魔法の絨毯に乗るような経験に仕立てている。これで話は大違いとなる。

ここまでにする。一応著作物なんであんまり長いのはちょっと

141 :
>>140

ありがとうございます

142 :
Japanese curry is my favorite curry. Just not the one with apples.

お願いします
Justがわかりません

143 :
not the one with apples
をピンポイントで強調してるだけ。
リンゴ入りカレーは大嫌い。

144 :
>>142
the one with apples

これ、リンゴが「丸ごと」いくつか入ってるという表現にならないの?
まあ、それでもいいが・・・・・・

the one with pieces of apple この表現との違いはあるよな。   

145 :
前文との文脈を考えれば「リンゴ入り以外はね」の意味に取れると思う

146 :
>>142
>>Japanese curry is my favorite curry. Just not the one with apples.

確かに、with appleS だったら、一皿のカレーにいくつかのリンゴが丸ごと入っている
という意味になってしまうな。つまりたとえば、1皿のカレーに3つくらいのリンゴが丸ごと入っている
なんておかしいよな。

そういう時には、確かに with pieces of apple でもいいけど、単に with apple でもいいのだ。
つまり、無冠詞で不可算名詞としての apple で、with apple だ。

さらに、not the one なんて言わなくてもいいんじゃないかな。というわけで、この英文を書いた人は
英語ネイティブではないような気がする。英語ネイティブなら、こういうときに appleS とは書かないような
気がする。さらには、Japanese ★curry★ is my favorite ●curry● というふうに、1つのきわめて短い
センテンスに curry を二つも使っている。ますますネイティブらしくないような気がする。
仮にネイティブなら、よっぽど文章の下手なネイティブだな。

俺がこの文章を書くとしたら、次のように書く。

Japanese curry is my favorite. Only not with apple.

ここで only とは、「ただし」というような意味。こういう only は、辞書では副詞じゃなくて
「接続詞」として扱われている。「ジーニアス」とか「ウィズダム」には、接続詞としての
only をきちんと解説している。他の辞書については、知らない。

147 :
ネイティブはa peace ofを省略してaって書くよ。省略して良い物、悪い物あるかもだけど。

148 :
↑apple slice

149 :
Curry with Apples, Kale, Chickpeas & Cashews By OhMyVeggies.com
Oh My Veggies › Recipes
ネイティブはふつうにApplesだね。

150 :
(1) When I first came I did chicken ★curry with apple and sultanas★ in it
and the children didn't like it all so I had to change the recipe.

(2) Middle row from left: Debito curry, Pace curry, Ash (game only character)
curry (Mild Japanese style ★curry with apple★).

(3) ... Seafood Fra Diavolo over basil orzo, Wild Mushroom Lemongrass
and Ginger with vermicelli noodles and ★Chicken Curry with apple and mint★ ...

英語ネイティブでも、英語をよく知っているネイティブと、知らないネイティブがいる。
2ちゃんねるの日本人たちが日本語を知っていると言えるのか?

151 :
1)Chicken Curry With Apples Recipe - Genius Kitchen
Genius Kitchen › recipe › chicke...

2)Chicken Curry with Apples - Dinner: A Love Story
Dinner: A Love Story › chicken-curry-wi...

3)Curried Shrimp and Apples Recipe | Taste of Home
Taste of Home › recipes › currie

4)Recipe for chicken curry with apples and onions - The Boston Globe
The Boston Globe › story


sliced 、slices を省略しただけでしょ。

152 :
リンゴをジャガイモみたいに使うんだろうな。

153 :
りんごをスライスしろといわれたら
イメージ輪切りから入る俺涙目

154 :
焼き肉のタレにもりんごとか梨を擦って入れるとうまくなる。

155 :
Keep it rolling. Pedal to the metal. Oh, and tell FOX to double your budget.

アメリカのFOXNEWSについてらしいのですが
翻訳お願いします

156 :
>>155
この1分前にツイートしてる、FOX放映のドラマ"The Passage"についてじゃないかな

突っ走り続けろ。アクセルを踏み抜け。そうだ、あとFOXに予算倍にするように言いなはれ

157 :
これってどう訳したらいいのでしょうか?

Would be nice if you talk to them and ask the owner whatsapp
直訳で
「もしあなたが彼らと話せて、ワッツアップのオーナーに尋ねることができたらいいな」

で合ってますか?

158 :
https://i.imgur.com/WdaojDy.jpg

すみません、送られてきたものを貼ります。
「owner whatsapp」のwhatsappはラインの海外版whatsappのことを言ってると思うのですがワッツアップの所有者と訳していいですかね?

それと結局はBOSSとCATのmarge(統合?)がしたいことを言ってると思うのですが合ってますか?

159 :
>>156
ありがとうございます

160 :
>>158
Would be nice if
if以下の事をしたら良い事があるかもしれません

161 :
1 I couldn’t watch Cohen today so can someone give me a 5 word summary?
2 Consigliere turns on mob boss.

2はコーエン(大統領顧問弁護士)がギャングのボス化している
という意味ですか?

162 :
>>160
ありがとうございます!

163 :
検索すると Consigliereは マフィアの顧問
トランプをマフィアと例えてる。
マフィア(トランプ)の(元)顧問は(格は下がったが)暴力団の親分になった。

164 :
turn on 〜
〜に牙をむく

165 :
>>163
ありがとうございます

166 :
>>164
mob bossはトランプなのね。
トランプの顧問してたが今ではトランプに牙をむく、トランプを攻撃してる。
ということなのか。

167 :
consigliere というイタリア語の単語については、The Godfather の Part 1 で何度も
使われているけど、その中でも特に最初の方の結婚披露宴の席で、Al Pacino が
婚約者 Dinane Keaton に対して consigliere の意味について解説している場面があるな。

168 :
consigliere とはマフィアの中でどういう仕事をしているかについて
1分45秒で簡潔かつ詳しくわかりやすく解説しているビデオを見つけた。

What is a Consigliere? ? Mafia Family Structures
https://www.youtube.com/watch?v=0tk15YHjamE

169 :
こんしりあーれ みたいに発音するのね。
ご意見番でイエスマンじゃないんだ。
コーエンはトランプのこんしりあーれだったがトランプを攻撃する側に回ったんだ。

170 :
A bunch of our gang are coming out early so we can fit as much as in possible.
日本行くのが楽しみだという文の次の行でした。
よろしくお願い致します。

171 :
俺たちすぐ日本に行くからすぐ日本になじむはずだよ。

172 :
>>171
早速どうもありがとうございます!
何度か日本に来ている方なのですが、fitはやはり馴染むと言うことなのですか、ありがとうございます!

173 :
>>170
うちらの一部が先乗りして、皆ができるかぎり適応できるようにするんだ
かな?(come outの意図がよくわからんし、もしこうならearlierのほうが適切だとは思うけど)
もしかしたら本当に文字どおり「早期にカムアウトする」という意味かもしれない
後半はfit in as much as possibleではないのかしらん

174 :
>>173
ご親切どうもありがとうございます!!
確かに、もしかしたら we can fit in で = 合わせられるよ
だと嬉しいなとも思うのですが、最初の部分も意味がわかりませんし…
イギリスの方なのも関係してるのでしょうか。。

175 :
>>173
日本に敬意を表して日本を中心にするから日本からみれば来る、comeでoutは強調。

176 :
>>170
>>A bunch of our gang are coming out early so we can fit as much as in possible.

(1) 正確にいうと、fit as much in as possible が正しい。

(2) "fit in" の in はもちろん副詞。"fit in" で、「(日本または当該の地域に)馴染む」という意味。

(3) ここでは、"fit in" の全体に as much as possible をくっつけるのではなくて、
in だけを as much as possible で修飾したいので、"as much in as possible" となるのだ。

(4) ただし、"fit in as much as possible" も許容されているかもしれない。

(5) "come out" は、"come out to Japan" というような意味合いを含んでいるから、out を
つけてある。come to Japan というよりも、out をつけた方が(うまくは説明できないが)
「わざわざそこまで(遠いところまで)行く(来る)」というような意味合いが出ると思う。
それはちょうど、たとえば聴衆に向かって「舞台に上がって来いよ」と言いたいときに
Come out to (または onto) the stage. とか言って、out をつけることもよくありそうな気がするけど、
それと同じだと思う。

あるいは、「イギリスから外に ★出て★、日本にやってくる」わけだから
out をつけるのだという説明の仕方もできる。聴衆に向かって言う台詞の場合も、
「聴衆の席から ★出て★、舞台に上がって来い」ということだから out をつけているとも言える。
いずれにしても、こういうふうに日本語訳には表れないような out もよくあるけど、英語圏の人間から
見ればごく普通の発想で out をつけたくなるのだろう。

177 :
>>170
>>A bunch of our gang are coming out early so we can fit as much as in possible.

(6) "A bunch of our gang" は、他の人が指摘してくれた通り、
「our gang のうちの何人か」という意味だろう。

(7) are coming out と進行形になっているのは、もちろん近未来だから。

(8) early は earlier とする必要はないと思う。「早めに日本に入る」わけだから
「早めに」という意味で early だ。もしも earier にするんだったら、

A bunch of our gang are coming out earlier than the rest. と
しなければならなくなるような気がする。

178 :
"A bunch of our gang"
多勢の仲間達

179 :
>>174
fitの意味直接聞いてみれば?fit inの事なのか、他の意味なのか?
文脈だと一部が先に行くからなるべく全員揃うよみたいな感じだけど

180 :
調べてみたらa bunch ofでひとかたまりみたいだね。て事は予定より早めにみんなが行くから日本に馴染めるね的な。

181 :
fitとかfit inて都合つけるとか時間をとるとかじゃなく?

182 :
そうだね

2) 〈人に〉日時を都合する[つける].
用例
I'm very busy but I can fit you in at 4:30. 私は忙しいのですが, 4 時半ならお会いするのに都合がつけられます.

183 :
みなさま、ご親切にご丁寧にどうもありがとうございます!!

野暮を承知で、この一文解らないのですが?と訊いたのですがまだ返答いただいていません。
日本はそれなりに知っている方々なので、fitが“馴染める”以外の表現だとしたら…とも考え
大人数で仕事に来られるので忙しいでしょうが、会えたら嬉しいですと前自分が書いたので
もしかしてそれに対して、会えるよと言ってくれてるのかなと期待もしてしまったのですが
やっぱり[仲間達が早く行くから〜]の意味するところも全く??難しいです。。

184 :
fitとかfit inて都合つけるとか時間をとるとかじゃなく?

185 :
仕事なのか遊びなのか知らないけど、
「なるべく色んな事ができるように早く行くよ」
ってことじゃないの?
inの位置が変なのがよくわからんけど

あと、そもそもsoの意味がよくわからんな
少なくともこの一文からはsoによる意味のつながり方がよくわからん

186 :
A bunch of our gang are coming out early so we can fit as much as  in   possible
inが要らない気がするな。
A bunch of our gang are coming out early so we can fit as much as possible
as much as possible できるだけ
の意味がある。
多勢で早く行くからできるだけ間に合わせることができるよ。
間に合わせる内容は文脈による。
ボランティアで救援作業とか。

187 :
>>185
so (前文を受けて) 〜であるから

188 :
>>186
>>185
182まで読めばすでに答え出てるじゃん。同じ事書かなくても。

189 :
>>187
んなこた誰でも知っとる
この一文でのつながりが方がって言っとる

190 :
俺は皆さんのレスを見てやっとわかったような気がする。

191 :
My wife is rightly pissed by headlines like this:
“Stephen King and his wife donate
$1.25M to New England Historic Genealogical Society.”
The gift was her original idea,
and she has a name: TABITHA KING. Her response follows.

翻訳お願いします

192 :
>>191
「スティーヴンキングとその妻がNew England Historic Genealogical Societyに約一億円を寄付」
こういう見出しに妻はうんざりしているが、無理もない。これは妻の発案であるし、妻にはtabitha kingという名前がある。妻の言葉を次のツイートに載せます。

193 :
>>192
ありがとうございます

194 :
I like your T-shirt The old one can i have it

これはどういう意味でしょうか?
そのまんま、あなたのTシャツが好きです、古くなったらちょうだい!ということですか?

195 :
日本語にない定冠詞のお勉強
your T-shirtの内のThe old one特定の古いシャツ。
The old oneを今ちょうだい!

196 :
>>195
なるほど、簡単に言ったら「それいいなぁ。ちょうだい!」ですね。
ありがとうございました。

197 :
'He told me that he didn't know I had acne like that. He said that I shouldn't lead people on like that'

お願いします

198 :
>>197
彼は私に、君にそんなニキビがあるとは知らなかったと言いました。君はそんなふうに人を引っかけるべきじゃないと言いました。

199 :
The place looks like great things to do.

ってどんな意味ですか?

200 :
ある場所に行かないかと誘ったときの返答です。

201 :
その場所に行くことはgreat things to doであるように思う

というような意味ですか?

上手い訳はありませんか?

202 :
その場所はそれをするのにちょうどいいみたいね

203 :
The place looks like great / things to do.

204 :
>>202
>>203

ありがとうございます。



私のおすすめは京都へ行くことと奈良へ行くことです。

みたいな英文に対する返答として、

Those places both look like great things to do.

が返ってきたのですが、

Those places both look like great
things to do.

と切ったとして、どのように訳せばいいのでしょうか?

205 :
あ、多分、分かりました。

私のおすすめは京都へ行くことと奈良への観光です。

のように訊ねたので、

things = sightseeings

だと思います。

情報が足りなくて申し訳ありませんでした。

ありがとうございました。

206 :
1 By the way, my Tesla is currently
getting 22,000 miles to the gallon. Mechanical problems? Zero.

2 And how much coal is burned to make this 'green miracle' move?

3 I get the juice that powers my Tesla from solar.

4 Let me amend that. We get a credit from Central Maine Power because we sell them solar.
It’s not a huge deal, but it’s a start.

翻訳お願いします

207 :
ゲーム内のセリフです
女好きの怪物トロールに関しての注意を
女キャラにしているという流れだと思われます

...They definitely will by the looks of you.
By the eight.. You'd better run as soon as you see trolls around you. Ok?

will by the looks of youとBy the eight..の部分が全くわかりません
翻訳お願いします

208 :
検索した結果...
They definitely will 攻撃してくるby the looks of you.
By the eight.危ない目にあったら. You'd better run as soon as you see trolls around you. Ok?
troll 
地下やほら穴に住む超自然的怪物で,巨人または小人に描かれる
the eight 
ビリヤードでは8番の玉を台のポケットに落としてはいけないというルールがあり、
8番ボールの後ろに自分がいて、それを打つとまずいことになるところから。

209 :
>>207

By the look of you = あんたの身なりからして(奴らは確実に襲ってくる)
By the eight = oh my god, damn itの意味

210 :
https://tvtropes.org/pmwiki/pmwiki.php/Main/OhMyGods

211 :
翻訳お願いします

I love Nick, he's my fave member. But I prefer Joe's voice is like... more clen? I understand better what he's saying. However I feel Nick has more control over his voice

212 :
>>208-209
ありがとうございます!
助かりました!
そして勉強になりました

213 :
>>211
私はニックが大好き。私のお気に入りのメンバー。でもジョーの声のほうが好き、
なんていうか、ニックより(声が)クリーンじゃない?
声がはっきりしていて、言ってることが聞きとりやすい。
でも、ニックのほうが自分の声をうまく制御できている感じがする。

214 :
>>210
By the Eightは正にこの中に挙げてあるゲームの言い回しでしたw
ありがとうございました

215 :
>>213
ありがとうございます!

大変恐縮ですが、こちらもお願いします🙏

ニックとジョー、どちらが歌唱力たかいのか?の返答です
Is that weird that i think Joe is the coolest one among them all

216 :
>>215
全員の中で、ジョーが一番クールだと思うんだけど、変かな。

217 :
Geekdom at its very best:
What do you think of the theory that
the “Mountain Kaiju” is actually Rokmutul?
This question may keep me awake tonight.

翻訳お願いします

218 :
.We should go out one night for a beer or two

飲みに行こうといっているみたなのですが、何だか
泊まりもあるし、怪しい誘い方なのでしょうか?
あとTWOの意味がわかりません(;´・ω・)何かお酒
の名前?? 

219 :
>>218
a~or twoで一つか二つ
Hありかどうか文脈と男次第

220 :
>>218
お茶しないっ?
と同じだと思うよ。

221 :
Put the new ink cartridge in the bottom half of the pen and then screw on the top.
翻訳お願いします。halfのあたりの意味が読み取れません。

222 :
新しいインクカートリッジをペンの半分くらいまで押し込み、その後ねじり切って下さい

223 :
>>221

新しいインクカートリッジをペンの下半分(先の方)に入れて、(ペンの)上側を回して締めて下さい。

ペンの説明で合ってる?

224 :
New phone. send through numbers

翻訳お願い致します。
send through numbers がさっぱりわからずです

225 :
ねじ切ってはいけないと思う。

226 :
https://www.youtube.com/watch?v=dqNT_mq0rwU
https://www.youtube.com/watch?v=9-88HxyUENk

227 :
>>222,223
ありがとうございます!
bottom half で下半分か、なるほど
テキストの問題文なのでこれ以上の情報はないです。

228 :
おねがいします
What is it about the book that many people are buying?

229 :
>>216
ありがとうございます

230 :
お願いします

While Nick and Joe's solo music careers never reached quite the height that the collective trio did in their prime Disney days,
it's worth noting that in the interim, the brothers became bigger celebrities than ever.

231 :
>>228
強調構文のwhat疑問文

本に関して多くの人が買っているのは何か?


232 :
>>230

ニックとジョーはソロミュージシャンとしてのキャリアは、ディズニー時代の絶頂期にトリオで達したレベルには到達できてないが、その間に彼らはこれまでよりずっと有名になったということは特筆すべきだろう。

233 :
以下の文はどんな意味ですか?

Guests and host families are free to link up in different groups.

234 :
ゲストとホストファミリーは自由に乱交できます

235 :
I've always wanted to know the quadrillionth digit in the expression for pi. I'm disappointed that it is zero, not that it'd matter had it been a different digit.

おねがいします

236 :
>>231
本はtheで特定されてる。
その特定の本(ベストセラー)はどうして売れてるのか?
だと思う。

237 :
英語と日本語の違いの大事なこと。
研究者英和中辞典より引用
sports・man
スポーツマン 《★【解説】 狩猟・魚釣りなどの野外運動を好む人
; ★【比較】 日本語の「スポーツマン」は athlete に相当することが多い》.
引用終わり

238 :
Is it just me or have Eggman's plans gotten a littel wierd?
翻訳お願いします。haveとgetがどう使われているのかわかりません。

239 :
>>228
その本、色んな人が信じてるけど、一体どんな点が受け入れられてるの?

240 :
>>238
ソニック:エッグマンの計画、すこし妙なことになってないか?

疑問形でhaveが前に出てる

241 :
>>217
絶好調なオタク「mountain kaijuがrokmutulだった説、どう思う?
気になりすぎて今日は寝れんかもしれん」

242 :
>>233
ゲストとホストファミリーは異なるグループの中で行動を共にしても構いません

>>224
携帯新しくなったから電話かけて番号教えて

243 :
>>235
ずっとπの千兆桁の数字を知りたいと思ってきたが、ゼロが出てきたのは残念
これが他の桁だったとしてもゼロという数字に意味はないだろうな

244 :
>>242

「ゲストとホストファミリーは異なるグループの中で行動を共にしても構いません」

ありがとうございます。

ホストファミリーが複数いるのですが、それらの異なるホストファミリーたちが一緒に行動してもかまいませんという
意味でしょうか?

245 :
>>235
πの1000兆目の桁が何なのかはずっと気になってたんだけど、それがゼロだったのは残念。まぁ、別の数だったらどうこうってことはないんだけどさ。

246 :
>>238
ひょっとしたら俺んとこだけの問題かもしれないんだけど、Eggmanの計画って何かおかしくなってね?

247 :
>>233
これはもうすこし前後の文脈がないとわかんないね。
ステイ前のマッチングのときの話なのか、ステイが始まってからの何かのイベントの話なのか、とかで意味が変わってくる。

248 :
>>228
あの本、良く売れてるけど、どうなの?
とか
みんな買ってるあの本だけどさ、あれってどうよ?
とか、
あの本、売れてるけど、あれってなんなん?
みたいな感じ。

What is xxx about?ってこんな感じでよく使うよ。

249 :
>>233
>>Guests and host families are free to link up in different groups.

俺が想像するには、すでにどのホストファミリーがどのゲストを迎えるかが決まってしまったあとは、
こういうイベントをいちいち催す必要性も低くなるだろうから、ここではまだそれが決まっていないのだろう。

そこで、ホストファミリーが仮に10家族いるとする。ゲストが20人いるとする。ゲストたちは自分を受け入れてくれそうで、
しかも気の合いそうなホストファミリーを探し、ホストファミリーもまた、同じように気に入ったゲストを探している。

ホストファミリーは、1家族当たりで1グループを作る。そしてゲストは一人ひとり、あちこちのグループに入り込んで
食事しながら話をする。ゲストはいろんな家族(つまりグループ)と行動を共にする。家族の方は移動しない。

まあ、家族の方も移動したってかまわないだろうけどね。

それにしても、質問者はなぜそこまでこだわるのかな。実際にそのイベントに参加すればすべてわかることになるし、
どのような結果になったとしても、大したことはないんではないかな?

英文を読んだだけでは、いろんな意味合いになると思う。

250 :
tell the people you love that you love em today

これだけです
よろしくお願いします

251 :
君が愛する人たちに、愛していると伝えなさい

252 :
>>251
ありがとうございます!

253 :
Hey gays whu have malachit

英語が苦手なトルコ人がチャットに書いてたのですが何て言ってるか分かりますか?
グーグルで翻訳するとちょっと同性愛者がいますマラキットと出ます

254 :
マラキットはmalachite マラカイト 孔雀石 くじゃくせき 
らしいよ。

255 :
孔雀石で分かりました、ありがとうございます

256 :
>>219

ビア1−2ハイみたいな感じですね。
有難う御座います。One Nightは流石にやばいです。

>>219

決まったお決まりのフレーズだとしたら
そんなに怖くないですね。有難う御座います。

知らないとそうは訳せませんでした(^^

257 :
256>>

One Nightは別に最初から一夜を過ごしましょうとい意味ではなくて、ビールを飲む話なので、ある日の夜になったら出かけましょう程度の意味。その後の展開はその時の成り行きによりけり。

258 :
>>241

ありがとうございます

259 :
An enumeration spells out item for item the useful elements of an existing class of data.

item for itemがよくわからない。。

260 :
入れ替えが許されるならば
An enumeration spells item out   item for the useful elements of an existing class of data.

261 :
>>259

S(An enumeration) V(spells out) M(item for item) O(the useful elements) M'(of an existing class of data).

SはひとつひとつM'のOをspell outする。

word for word
で調べてみ。

262 :
protocol visit

の意味を教えてください。

263 :
Last valid date of JR Free Pass should be utilized for shopping in central Tokyo

の意味を教えてください。

264 :
JRフリーパスの有効期限の最終日は東京での買い物に充てるべきだ。

265 :
>>264

ありがとうございました。

266 :
>>247
>>249

ステイが始まってからの話です。

その日はゲストとホストファミリーの自由な時間の日です。他の日はいろいろ予定が詰まっています。

267 :
ホストファミリーとゲストは異なるグループの間で合流するのは自由です。

みたいな訳はどうですか?

268 :
>>262

市長に会いに行くらしいのですが、訳は表敬訪問みたいな感じですかね?

269 :
英訳をお願い致します。

・ドミトリールームの下段をお願い致します。

・ドミトリールームの下段ルームで寝たいです。

270 :
>>269
スレ違いですよ

271 :
take this higherを意訳するとどういう意味になるのでしょうか?

272 :
>>261
遅くなりましたが、ありがとうございました。

273 :
See you tomorrow
て明日初めて会う相手に対して使える?

274 :
There’s a difference between your (possessive) and you’re (short for you are).
Only president in history
who fell out of the dumb tree and hit every branch on the way down.

翻訳お願いします

275 :
Also, you've run that joke so far into the ground, it's gonna hit the US.

お願いします。
冗談使い過ぎだと言いたいのはわかるのですが・・・

276 :
>>274
あなたが何者であるか(you areの省略形)と、あなたのもの(所有型)の間には隔たりがある。どん底から這い上がった歴代の大統領だけが、この隔たりに存在するするあらゆる邪魔を取っ払う

277 :
>>275
また・・、○○がアメリカに衝突する、という感じの冗談をこれ迄言い過ぎだよ

278 :
Yourとyou’reは同じものではない
これだけ頭の悪い大統領は歴史上類を見ない

そのジョーク使いすぎ、アメリカまで届いちまうぞ(run into the groundと掛けている)

279 :
ありがとうございました。

280 :
He received a large reward, which was no more than his due.

コンマ以下がよくわかりません
彼は多くの報酬を受け取ったけどその報酬は彼には多すぎる?少なすぎる?ちょうどよい?

281 :
>>280
彼は多大な報酬を受けたが、それは彼が当然受けるべきものに過ぎなかった。
..., and it was just as much as his due.

282 :
which was no less than his due
だとどうなりますか
意味が通らない文章になってしまいますか

283 :
>>282
彼は多大な報酬を受けたが、それは彼相応の分として不足はなかった。

284 :
多大は多すぎたので訂正。
文脈によって可能な訳としては

彼は大きな報奨を受けたが、それは彼のはたらきに対して[多すぎる(more) / 少なすぎる(less)]ものではなかった

285 :
ある本の紹介の一部なのですが、

The scene is one of some interesting things the book offers up with a regularity that is almost boastful in its success.

そのシーンはその本が定期的に提供するいくつかの面白いことのひとつである。

that 以下はどのように訳せば良いのでしょうか。お願いします。

286 :
Truer words never spoken.
Hard to believe any armed services vets can continue to support
the draft-dodger president.

翻訳お願いします

287 :
絶対語られない真実の言葉なんだけど、
軍の軍人が腐ったれ大統領を支持し続けるというのは信じがたい

288 :
>>285
その本が定期的に提供する面白いことの
一つなのである。そしてそれがその本の
成功をもたらしていると言っても過言
ではないのである

289 :
お願いします。

Back on my 3yr old MacBook Pro after trying a new model for 3 months.
Such a relief of not having to cope with the broken keyboard and touch bar anymore!

Google翻訳だと
「新しいモデルを3か月間試用した後、私の3年前のMacBook Proに戻ります。
壊れたキーボードやタッチバーに対処する必要がなくなりました。」
ってなるんですけど、前のモデル(おそらくキーボードが壊れてる?)に戻るけど対処する必要がなくなる??となって混乱してます。

290 :
>>288
ありがとうございます。

291 :
>>289
壊れているのは新しいモデルだろ
きっとだましだまし使ってたけど我慢ならなくなって前に使っていたやつに戻したんだ
まあbrokenといっても物理的に壊れるだけじゃなくて
思ったような動作をしてくれないという可能性もあるよ

292 :
>>291
ああ、そういうことか……前後の文脈がわからなかったので、勝手に「古い=壊れてる」と思いこんでました
ありがとうございます

293 :
>>285
そのシーンは、その本がその成功においてもはや鼻につく程の定期性を伴って現れる、数ある面白いシーンのひとつである。

a regularityは名詞なので、that以下はその説明と考えてみた。

294 :
お願いします

There's no denying that things have changed in the six years they've spent apart. Joe's solo career never truly took off,
but he did find some measure of success with his band DNCE. Nick found a bit more musical success as a solo artist,
and also some modest success an actor, ready to step into whatever role Zac Efron's schedule can't accommodate.

295 :
>>287
ありがとうございます

296 :
>>293
なるほど、そういう捉え方になるのでしょうか・・・
自分では>>288さんの訳とどちらが適切なのか判断できませんが、ありがとうございます。

297 :
服に書いてある英文の訳をお願いします。
This is not a 以降の筆記体が読めないので商品ページを貼らせてもらいます。

ttps://item.rakuten.co.jp/reca/100406/?s-id=bh_sp_item_image

298 :
mirageに見えるけど自信はない

299 :
>>294
彼らが離れて過ごすようになった6年で、状況が変わったのは否定しようのないことだ。ジョンはソロとしての仕事は全く軌道にのらなかったが、自分のバンドであるDNCEに成功の糸口を見出した。

ニックはソロミュージシャンとしての音楽的成功まであと一歩まで近づき、ザックエフトンが多忙のため演じれない役ならなんでも代役になれる程度にはささやかな役者としての成功にありついた。

300 :
>>299
ありがとうございます!

301 :
 できればこちらもお願いします🙇


If this was any other photo then perhaps it wouldn't warrant so much conjecture.
But it was chosen to signal the return of the band and set the agenda for a new era of the Jonas Brothers.   
There is no other way to read it than that Nick is now most definitely the focus of the band.

302 :
Ask any group of children in Japan what they want to be when they grow up and one answer in particular is sure to feature.

お願いします
最後の方ですね

303 :
>>301
これが他の写真なら、いろんな推測をもたらす余地もないはず。
だけどこの写真が選ばれたのは、バンドの復帰を告げるため。ジョナサンブラザーズの
新しい時代の到来を告げるためなんだ。
ニックがバンドの焦点だという以外に読み様がないわね

304 :
>>302
日本の子供たちに大人になったら何になりたいか尋ねてみて。
1つの答えがとりわけ重要であるはず

305 :
>>298
ありがとうございます

306 :
According to my "365 Facts That Will Scare the Shit Out of You"
daily calendar, Slumber Party Barbie once came
with a book titled HOW TO LOSE WEIGHT.
The visible text reads, "Don't eat."

翻訳お願いします

307 :
>>306
おれっちがもってる日めくりカレンダー「あなたをゾッとさせる365の真実」によれば、
昔、パジャマパーティー・バービー人形には『減量法』という本が付いてきた。
その読めるところの文には「何も食うな」と書いてある。

308 :
>>306
私が持っている"365 Facts that Will Scare the Shit Out of You"という日めくりカレンダーによると、
体重の減らし方という本を手にしたパジャマパーティー姿のバービーが登場したことがある。
その本に書いてある文字は「食うな」だ。

309 :
He has the professional element to deal with ,as well,now, which is more difficult to outsmart than are the members of Group Two.
お願いします。

310 :
>>309
今も同様にやつのプロフェッショナルなところをなんとかすることになるのだが、
それをうまくやりすごすのはグループ2の連中とよりも難しい。

311 :
>>302
日本の子供達(人種性別などを問わない)に将来何になりたいかを尋ね、回答の一つを掲載します。

>>309
そうなると、専門性を要する分野に対応する必要も生じることになり、第2群のメンバーと比べて裏をかくことが難しくなる。

312 :
I don't know what you consider.

あなたが何を考えているのかわからない。
あなたの考えていることがわからない。

これどっちでもいい?

I don't know what you think.

なら、
あなたが何を考えているのかわからない。

I don't know what you think about.

なら、
あなたの考えていることがわからない。

で合ってる?区別なくても良い?

313 :
有難うございました

314 :
「そのためお互い嫌いになろうとしています」

英訳頼みます

315 :
>>307
>>308

ありがとうございます

316 :
>>312
疑問代名詞か、関係代名詞かってことなんだろうけど、最初の2つの日本文は意味が等価に見える

印象論になってしまうが
I don't know what you think about.
は「あなたが何について言っているのか(何を念頭に置いているのか)わからない」というふうに聞こえます
細かく言うと、述語はわかっても主語がわからない状態
それに対して
I don't know what you think.
は皆目わからない状態

317 :
>>316
やっぱり日本文は区別しなくても問題ないですよね
なるほど、aboutありなしのニュアンスも納得です

こんなわかりにくい質問に答えていただいてありがとうございます

318 :
vote presentの意味がわからない。 vote for , vote against以外にvote presentというのは
初めてみた。 だれか教えてください。

How embarrassing is the green new deal?

So embarrassing that when Senate majority leader McConnell tried to force
the Democratic party’s presidential contenders into an embarrassing vote over
the berserk, MMT-inducing climate-change proposal (which Republicans are
confident that even sober liberal will oppose), not a single Democrat voted for it.
Instead, in the vote which was blocked late on Tuesday with a vote of 0-57,
43 Democrats voted merely "present", including the Senate’s half-dozen
presidential candidates, to sidestep the GOP maneuver and, as Bloomberg put it,
"buy time to build their campaign positions."

319 :
>>318
https://www.conginst.org/2013/04/02/voting-present-as-a-legislative-tactic/
When a bill or amendment is up for a vote, a Representative may vote “aye”, “no”, or “present”, which is a refusal to take sides.
A “present” vote does not count towards or against the passage of a bill, but it contributes towards the quorum, which is the minimum number of Members required in attendance for the body to conduct business legally.
採決にあたって賛成か反対かは表明しないけど、採決に必要な出席数にカウントされるらしい

320 :
>>319

文脈的にもこれで意味がとおりますね。疑問がはれてスッキリしました。ありがとうございます。

321 :
I mean, if I died, the only way people would even know that I was here would be by the ass print on this chair.
お願いします。
would beの後のbyの意味がわかりません。

322 :
by (seeing) the ass print on the chairと同じことでは

323 :
椅子に付いた尻の跡によってのみ自分の存在を推し量れるだろう

〜によって

324 :
I'm the Welsh, but I like Korea.

325 :
I'm an Irish man, but I like china.

326 :
>>318
against・・・向かい風のこと。
Against wind.

327 :
Democratic party・・・これは中国共産党のことかな?

328 :

Democratic party・・・民主党・・・オバマさんかヒラリーさん
Republican party・・・共和党・・・トランプさん

329 :
>>321
手段 方法の by じゃないの wayと対応してるし

I mean,
if I died,
the only way people would even know that I was here
would be by the ass print on this chair. 

330 :
はいはい、これは簡単。presentはそもそも出席とか現代って意味があるよね。
まず、ジャンルの理解だね。だいたい時事ネタ、ニュースネタが多い。
ニュースはまずは、政治経済だよね。
deal/vote/climate/Democratic party/Republicansってことから、これは政治の分野であり、
投票や選挙に関する話だと分かるね。

"Senate majority leader McConnell"がポイントでSenateは米国の上院。
上院議員のミッチ・マコーネルさんの話である、と分かる。

>How embarrassing is the green new deal?
>So embarrassing that when Senate majority leader McConnell tried to force
>the Democratic party’s presidential contenders into an embarrassing vote over
>the berserk, MMT-inducing climate-change proposal (which Republicans are

331 :
ついでにアメリカの政治つーか議会用語を覚えよう!
新聞社の政治部の人くらいで、普通の日本人は全く要らない知識。

・米国の立法府について
the House:議院・・・連邦議会
a member of the House:議員
上院(the House of Senate)
下院(the House of Representatives)(or Congresses?)
a member of the House of Representatives:下院議員
a Congressman:下院議員
a Congressperson:下院議員

332 :
次に選挙だから、presidential candidates・・・って後に出てくるからな。
presidential contenders・・・大統領って頭についているから、大統領候補ということだね。

ちなみに大統領補佐官はAssistant to the President for National Security Affairs
White House Chief of Staffがアメリカ合衆国大統領首席補佐官
ってことらしい。いかにもTOEICに出て来そうな用語だ。

333 :
政治だから、次は政策の話になるが、これはアメリカの現代経済学の話になる。
MMTとキーワードが出てくる。これは経済学部の学生か理系なら経営工学関係しか、
分からないよね。MITなら大学なんだけどね・・・
Modern Monetary Theory・・・現代貨幣理論とは経済学の理論で、不換貨幣理論と呼ぶらしい。
ここでいう不換貨幣とは、「政府発行紙幣」という時事用語らしい。

既に、英語の問題というよりアメリカの政治と金融経済の話だと分かる。

334 :
the green new deal・・・これにはヒントが二つある。グリーンだから緑=自然に関すること、
new dealだから、政策、ニューディール政策?ということなんだけど、
この二つを組み合わせた、造語じゃないかなと、固有名詞っぽいんだが、一般的に
書いてある。the Green New Dealって書いてくれれば、完全に固有名詞だからね。

>『グリーン・ニューディール』は、2008年7月21日にグリーン・ニューディール・グループが発表し、
>新経済財団により出版されている報告書、もしくはその内容に沿った政策の名称である。
>地球温暖化、世界金融危機、石油資源枯渇に対する一連の政策提言の概要が記されている

335 :
しかし、内容は英文読解だから、これらの政策や経済用語が分からなくても大丈夫だ。
・グリーンニューディール=地球環境関係の政策?くらいにあてずっぽうで理解
・マコーネルさんが、政策関係で民主党の立候補者に圧力を掛けている?
(アメリカの選挙の投票日が火曜日なので、チューズデーと入っているが、どうでも良い)

↓メインの内容はここだけ。
Instead, in the vote which was blocked late on Tuesday with a vote of 0-57,
43 Democrats voted merely "present", including the Senate’s half-dozen
presidential candidates,

336 :
43+57=100だから、100%なのか100人なのか?ってことだけど、
blocked late on Tuesday with a vote of 0-57
43 Democrats voted merely "present"
including the Senate’s half-dozen presidential candidates,
→上院の半ダース(6人)の大統領候補を含んで、
民主党員43人が投票したんで必要な議席数を満たしたってことかな。

337 :
英語初心者でセサミストリートでリスニングと英文勉強してるレベルです
エルモが子供とアニメのキャッチボールシーンを見ながら
Boy,you're going to really catch it now.

エルモが金魚の友達に耳を近づけてうんうん頷きながら
Want Elmo to tell.
なんとなくエルモに行って欲しいことがあるんだね うんうん 的な意味合いだとは思うんですがあってますか?
この場合の冒頭Wantは命令形?という意味合いではなく子供の話し言葉としての意味合いが強いですかね?

338 :
(You)Want Elmo to tell(something).
単に省略って気がする
命令というか 〜だね?って感じがする

339 :
英語も日本語と同じで主語がなくってもいいんだな

340 :
>>337
Boy, you're going to really catch it now.
わあ、今から実際にキャッチするんだね

(Do you) Want Elmo to tell it?
教えてあげようか?

こういうのは映像を見ないと正確なところはわからないよ

341 :
>>340
https://youtu.be/3gt6cHooI6M
Boy,you're〜 2:47(2:40あたりから)
Want Elmo〜 1:50(1:43あたりから)です

Boy,you're〜は流れ的にしっくりきますね!
この場合のyouは男の子ではなくアニメ内の人物について表してるということですかね?

>>338
なるほど シーン的に主語省略のこの意味合いが近いかもしれないです!

342 :
Can you read this news?

조금씩 따뜻해졌습니다

343 :
>>341
見てみた

> youは男の子ではなくアニメ内の人物について表してる

そういうことみたい

2つめは、語尾も上がっているし口語でよくあるDo youの省略だろうな
金魚のドロシーはなにか質問したいことがあるけど、当然話せないので
「エルモが(代わりに質問を)言ってあげようか?」
みたいな流れだと思う

344 :
自分が意訳するとこんな感じ

boy 間投詞 驚き
really 強調 
be going to 意図 するつもりである しようと思っている
now 今すぐ 今や

わあ本気でキャッチしたいんだね


Dorothy’s been thinking about balls, too.
And Dorothy has a question
Want Elmo to tell it?
OK.
How do you catch a ball
Good question, Dorothy.

(Do you) want Elmo to tell it? エルモが言っていい?
it = the question

345 :
Your things are mine, my things are mine too.

346 :
ビッグGキタ━━━━━(゚∀゚)━━━━━!!!

347 :
>>343
>>344
字幕オフにしたら Want Elmo to tell it?とクエスチョンマークが付いた質問文だったので省略形で間違いなさそうです
ありがとうございます、謎が解けました!

348 :
Release the Mueller Report and have done with it.
Quit the games.
THE AMERICAN PEOPLE PAID FOR THAT THING.

翻訳お願いします

349 :
トランプいい加減にしろ

350 :
>>348
ムラーの報告書を公開してこの件にケリをつけろ。
ああだこうだはおしまいにしろ。
アメリカ国民はそのための金を出したんだから。
(訳注・トランプ陣営のロシア疑惑に関する調査報告書。
議会にはその要約のみが報告されている状態)

351 :
>>350

ありがとうございます

352 :
インスタに有名人のツーショット画像をあげました
目がきらきらしているとコメントを添えました
それに対する返信コメントです

Thats not 〇〇〇(画像の片方の個人名)bud.

直訳するとそれは〇〇〇の芽ではない、ですがニュアンが解りません
どんな感じのことを言いたいのでしょう

353 :
Bud=buddy=friend

354 :
>>353
ありがとうございます
つまりこの場合のbudは〇〇〇にかかってるんではなく、よーブラザー的な私に対しての呼び掛けですね?
インスタの内容は確かに男性的ですが私自身は女なのでレス貰ってからもしばらく考えこんでしまいました

355 :
I can't be controlled. So all want is someone who can watch me do my thing and say "that's my baby"

よろしくお願いします。
外国人の親友が心の病気になって、誰も連絡がとれていません。
そうしたら、めったにSNSに投稿しないのに、その子がこの文章を投稿しました。
これは助けがほしいということでしょうか?
それとも何かのジョークでしょうか?
ググッたらどうやら有名な文章らしいのですが、全て英語なので理解できませんでした。

どうぞよろしくお願いします。

356 :
your tm8 saved ya lol gg tho
ゲームをしていたらこんなメッセージが来たのですが単語を調べてもこの文章にどんな意味があるのか分かりません、分かる方いませんか

357 :
>>355
私はどうあってもコントロールされない。だから私がほしいのは、私が好きなように行動するのを見て、それでも「あれが私のベイビーだ」と言える人だけ。

358 :
>>356
君のトマト?に救われたね(笑)でもいい試合だった
(自信なし)

359 :
>>357
ありがとうございます。
本当にありがとうございました。

360 :
>>356 >>358

横レスだが、tm8はteammate だろうね。

361 :
>>358
なるほど good game though か

362 :
こんなゲームを作ってるんだけど、

この、てきとーすぎる英語をなおしたいです

これでぜんぜん良いのかもしれないけど、良い表現があったら教えて欲しいです

https://i.imgur.com/CRD8fks.jpg

363 :
左の日本語をそれなりの英語にしたいです

364 :
The passing of May's deal, after two historic defeats,would be less
a sign of her skill negotiating tactics than of her running down the clock.

お願いします。意味や構文がわからない。

365 :
>>364
メイ首相の法案が通ったといっても、2回の歴史的な敗北の後では、
首相の交渉戦略の妙を示すものというよりか、時間切れに追い込んだ
ということ表すものだろう。

366 :
>>365
364です
再度お願いします
her skill negotiating tacticsの部分がどうもわからない
skillはskilledとかskillfulの意味ですか

367 :
>>364
her skill (in) negotiating tactics
inが抜けている気がしてならない
あとrun down the clockについては
https://ejje.weblio.jp/content/run+down+the+clock

2度の歴史的大敗の後で、メイ首相の提案が可決されたとしても、これは
首相の交渉戦術面での技量の表れというよりはむしろ、首相が時間切れ間際に守勢を固めていることの表れということになろう。

368 :
なんだろな
英 俗語 で skill and brill (すげえ)って表現があるらしい
brill 形容詞 = terrific
英国の人がかいたとしたら
skillを形容詞 すごい として使ったとか? 辞書だとskillは名詞ってあるけど

369 :
That is bullshit you know I kill you first but good luck they give it to you because you used the best

Do you know I'm Chi your first you know that but it's OK it's part of the game you win or you lots I'm no you're good trust me I know you th

ネット対戦で相手を負かしたらメッセージが届きました。意味を教えてください。

370 :
英国人のBILLY何とかさんの記事。
「SKILLはイギリス的表現ですか?」
とメールでも送ってみようかな。

371 :
You can care about that,if you really want me to come, cause i don't think that it's right if i pay for all on my own,my parents won't understand that

ウクライナ人女性との会話
本当に理解できない。
どなたか教えてください

372 :
私に本当に来て欲しいなら、きちんとその事を考えて下さい。
全部私が払うのはおかしい。
私の親もきっとおかしいと言うでしょう。

373 :
ありがとうございます。
助かりました。やっと理解できました

374 :
その文章を読んだらこれを思い出した
ジジイとデートした女性が家に招かれた日に片付けさせられたって怒るシーンがある
どうしようもないクソジジイなので中々見物
https://youtu.be/fpjq7KADFSI

375 :
Are you surprised that rich parents are bribing their rich kiddies
into top schools?
Shocked, even? Don't be. In the Age of Trump,
when even fundamentalist Christians wink
at the president's flagrant adulteries,
such behavior comes to seem normal.

翻訳お願いします

376 :
雑に訳すとこんなとこか

金持ち連中が賄賂で自分とこのキッズを一流の学校へ入れてるって聞いて驚いてるって?
ショックを受けるほど?そんなことしなくていい。

原理主義のクリスチャンたちでさえ 
大統領の目に余る不義に目をつむるトランプの時代だと、
そんなこと 普通のことに思えるようになってんだから。

377 :
>>376

ありがとうございます

378 :
素敵なことがいろいろできそう
なところだね

379 :
I recently i got out of a relationship and i was hitting up random guys here lol...so u single? r u into red.head chicks?

なんかゲームしてたらメッセージが来てちょっと拙いながらも相手と会話してたら女性の自撮り画像付きでメッセージ来たのですが翻訳してもらえませんか?

380 :
最近カレシと別れたから手当たりしだいに声かけてたんだけど、独り身?赤毛の女の子はは好き?

381 :
>>380
こういうのってなんか怖いし英語も読めないし無視しとくかな…
ありがとうございます、助かりました!

382 :
Osss

外人さんのやりとりでこの英語を時々目にするのですがどういう意味ですか?

383 :
As headstrong as her young charge, Ella did not normally alow herself to be overruled.

子育てに関する文です
分かるような分からないような感じなのでお願いします

384 :
>>383
年端もいかない子がききわけがないからといって、
好きにさせるなどということをEllaが許すなど
通常はなかった。

385 :
>>384
ここでのchargeはどういう意味?責任?

386 :
>>385
保護の対象者。
例えば、子供たちをつれてどこかに引率したりするとき、
その対象者をChargeという例がある。

387 :
>>385
保護の対象というか、
責任をもつ対象といったほうがいいかもしれない。

388 :
>>385
ツケ みたいな意味かな
「若い頃好き勝手やんちゃしたツケもあって、エラは普段から自分を自制していた。」

389 :
>>387
そんな意味あるのか
ありがとうございました

390 :
ちなみにエラを困らせたこの子はマルコムX君ね

391 :
あそっか、これ不可算のchargeか
>>387が正しいね

392 :
>>389
参考までに、こんな使用例がありますよ。
Chaperones were expected to exercise stern authority over their charges, this, combined
with the fact that young people typically had little or no say in the choice of
a chaperone, could lead to resistance and resentment on the part of
the young people being supervised.

393 :
>>392
わかりやすい例だ
chaperoneって単語も勉強になった

394 :
>>393
おもしろいでしょ。
どうしてこうなるかは、例えば、こんな英文
He is in my charge
とかを考えてみれば明らかですよね。

395 :
>>383
Ellaはベビーシッターか何かなんだろ

Ellaは受け持つ子供と同じくらい頑固で、普通は自分のやり方を曲げることは無かった

396 :
>>395
文頭のas 形容詞asの訳し方はどうなんだろう
これもいくつか文脈で意味が変わるのかな

397 :
>>396
As 形容詞 as といった言い方が文頭にきたときは
くせもので、文脈によって意味がまるっきり変わったりします。
下記urlが参考になるかも

http://blog.livedoor.jp/yamajun1985/archives/49759517.html

398 :
こういうの日本語であーだのこーだの言われればわかるけど
英語だけの人達はどうやって理解してるのか不思議でしょうがない………

399 :
>>398
たくさん英文を読めばわかるようになるといわれて
自分なりに腐るほど読みました。辞書もほとんどひかないで
ただひたすらに。
そうしたら、少しはわかるようになってきたけど、
いまでもとんでもない勘違いをしでかして恥をかくこともおおい.
いつまでたっても終わりはないね。
過程が楽しいからそれでいいけど。

400 :
(Being) As headstrong as her young charge (is headstrong),
Ella did not normally alow herself to be overruled.

headstrongからして 分詞構文(理由)だろうか

401 :
結婚する人に対するコメントでwsssって書いてる人が大勢いるんだけど、これは何の略?

402 :
>>383
エラは、面倒を見ている子と同じぐらい頑固だったので、発言を封じられる・変更を強いられるようなことは通例良しとしなかった。

403 :
文頭のas 形容詞asは逆説でずっと読んできたけど順接もあるんだね知らなかった
逆説じゃないよな?って思って質問したから納得しました

404 :
YOU: Interesting adaptation of the Kepnes book.
What's fascinating is the contrast between Joe's relationship with Beck
(inward and neurotic) and his relationship with Karen,
which seems more outward and adult.

翻訳お願いします

405 :
>>404
あなた:Kepnesの本の興味深い翻案。
すばらしいのは、ジョーとベックの関係(内向きで神経質)と、ジョーとカレンの関係(こっちはもっと外向きで成熟しているように見える)の対照だ。

406 :
英文翻訳スレに注意して下さい。
右翼等の反社会的勢力の工作員(ステマ業者含む)が
妙な文章を翻訳させる手口にて、変な文章を書かせる可能性があります。

胡散臭い英文は、注意し、翻訳しないで下さい!胡散臭い歌詞もありそうです。

適正なニュースソースの時事問題については
タイトルが明確な上で時事ネタスレを活用下さい。

407 :
I believe that the invitations should be sent by one who has more free time, rather than myself, who does not have much time to spare.
This is because I fear that there would be the possibility of our...

一緒にゲームをしているインド人からのメールです
翻訳お願いします

408 :
招待は暇な人にしてもらってくれ、俺は忙しいから無理

409 :
Your request well noted and added in your reservation.

We will try our best to arrange for you, but there's no guarantee at moment, but subject to availability upon arrival.

翻訳をお願い致します。

410 :
>>409
あなたの要求についてはたしかに銘記し、あなたの予約事項に付け加えました。
あなたの意にかなうよう私どもは全力を尽くしますが、現時点では何も保証することはできかね、ご到着時に利用可能かどうかに左右されます。

411 :
If you want people to say good things about you, it helps if they are expecting the question before the new employer calls.

よろしくお願いします。

412 :
There could be another rise in the price of petrol soon. (= It is possible that there will be.)

お願いします。文法書のcouldについての説明で、前後の文章はありません。
もうすぐガソリン価格の別の上昇が起こるかもしれない?
それは可能thatそうなるかもしれない?

413 :
>>412
「別の」というより「もう一回」

414 :
>>410
翻訳して頂きまして、本当に有り難う御座いました。
本当に助かりました。感謝致します。

415 :
couldの説明として
( )内は映画「今そこにある危機」みたいな言い回しで
そこにあるであろう可能性
ってことなんじゃ…
値上げがある(かもしれない)より
(あるでしょう、ありそう)だよという

416 :
There could be another rise in the price of petrol soon.
= It is possible that there will be (another rise in the price of petrol soon).
じゃないの

417 :
>>413
>>415
>>416
ありがとうございました。

418 :
They got themselves in solid.

they 火星人 で 訳やつらの立場はびくともしない になってた
注釈なしなのでなんでこの訳になるかわかりません

なんでや 出典はThe War of the Worlds

419 :
とりあえずの自分なりの解釈
get oneself in solid  = oneselfをin solid positionにした

420 :
>>418
グーテンベルクプロジェクトのテキストにはその1文がないね
リライトかなんか?

421 :
>>420
オーソン・ウェルズ主演のやつ 例のCBS放送 (1938)
https://www.youtube.com/watch?v=oWD9Q6klzco(47:44)

422 :
fak u sttuppid Japanese

Disgusting u r play like u r motherless

唐突にゲーム内で言われたんやが

423 :
fak u sttuppid Japanese

Disgusting u r play like u r motherless

唐突にゲーム内で言われたんやが

424 :
>>423
アホな日本人死にやがれバーカ笑

お前の育ち悪いようなプレイングめっちゃキモいわ笑

425 :
>>424


426 :
>>419
inは状態を表すinですね

427 :
>>421
solid(名)はcountableなので、このsolidは形容詞と考えられる

They got solid.
彼らはsolidになった。
They got themselves solid.
彼らは自らをsolidにした。
They got themselves in solid.
彼らは自分たちをin solidにした。

solidはメンバー個々の性質。
in solidは集合のありさま。米表現ではin solid with 〜で「〜と仲がいい」
これでは意味が通じないので、「関係が密、すき間がない」という原義に立ち返って
「奴らは自分たちをすき間のない、みっちみちの状態にした」

文脈に目を向けると、ここでは人類の生き残りが僅かであることと、敵戦力が豊富なことが対比されている。;(セミコロン)以下はその判断理由を表すと考えられる。
「奴らは世界最大の国を壊滅させた。例の緑の星は、おそらく毎晩どこかに落下しているだろう。奴らが失ったのは1機だけだ」
これを加味して
「奴らは地歩を固めた」

428 :
>>421
例の、勘違いしたリスナーがパニック起こしたというやつだよねwたしかにこれ臨場感あって、おもしろいね
時代的に生放送なんだろうか…だとしたらすごい
リスニング教材としてもよさげだし、こういうの他にも知ってたら教えてほしい

429 :
>>426, >>427
レスどうもです 参考になります

inはそんな感じがします
ただinが前置詞なので 原則としてうしろに名詞であってほしいですよね

形容詞だとDENSE系 STRONG系 それっぽいのあるんですが… 以下はSTRONG系
A structure that is solid is strong and is not likely to collapse or fall over コウビルド
で名詞形がsolidity (N-UNCOUNT) the solidity of walls and floors

んー ひょっとして
in solid.のsolidを堅固 堅牢 強固みたいな意味で非加算名詞としてつかったんでしょうか…

コンテクストからするとアメリカという地球上で最強の国を簡単に倒してて、このセリフの直後でも言及されてるので
意訳すると やつらは強大な敵だ ぐらいでもいんでしょうかね
まあgotは過去形なんですが このセリフの人物けっこうhave gotのhave省くんですよ
たとえば少々先にI got a planって言います

430 :
>>428
それですね メディア論とかに取り上げられるやつです

ユニコムって会社が出してた 全米ラジオドラマ傑作選 の1つをAmazonで買いました
中古でなぜか安かったし(配送料なしで178円、込みでも500円以下、パッケージによると2400円+税で売ってたそうです)
ミステリー読んで(聞いて)みたかったのと 少し気になってた作品だったんで買ったんですが 
中身は 訳と簡単な注釈の小冊子一冊とCDだけというシンプルなものでしたね

今日で全部聞き終わりましたが音声はYouTubeのほうがややクリアなぐらいです
正直自分のリスニング力だときつかったんで スクリプトみながら聞きました
複雑な構文や難しい単語が使われる箇所もちょくちょくありました 破格な文もけっこう多いです

初級
遠山顕の英語劇場 英語学習者向けの教材ですね
Dramas from BBC Learning English(やさしい英語で書き直したもの)
http://www.bbc.co.uk/learningenglish/english/features/drama/aliceinwonderland-ep01
あとはgraded readersがYouTubeでスクリプトつきで公開されてたりしますね
初級 中級
http://aswanpro.com/ 著作権ぎれの映画にスクリプトと訳をつけたもの YouTubeでも公開
イングリッシュトレジャー(語学春秋社 ウェブで視聴可能)
上級というか時事英語
NPR VOA GUARDIAN JAPAN TIMESなどのサイドにスクリプト付きの音声があるようです

431 :
>>405

ありがとうございます

432 :
>>429
名詞のsolidをそういう意味で使ってる用例は辞書には見当たらなかった
そこでget A in B という枠組みでなくget A (in solid)という枠組みで考えてみたんだが…苦しいかな
have got → gotはアメリカ英語ではありがちな傾向だと思う

ちなみに原文該当箇所:
“It’s all over,” he said. “They’ve lost one—just one. And they’ve made their footing good and crippled the greatest power in the world. They’ve walked over us. The death of that one at Weybridge was an accident.
And these are only pioneers. They kept on coming. These green stars—I’ve seen none these five or six days, but I’ve no doubt they’re falling somewhere every night. Nothing’s to be done. We’re under! We’re beat!”
http://www.gutenberg.org/files/36/36-h/36-h.htm

ラジオ版:
STRANGER: They got themselves in solid; they wrecked the greatest country in the world. Those green stars, they're probably falling somewhere every night. They've only lost one machine. There isn't anything to do. We're done. We're licked.
http://www.sacred-texts.com/ufo/mars/wow.htm

433 :
>>430
ありがとう
古い映画も名作揃いでいいかも

434 :
We want to offer you quick assistance with your request.

For payment-related questions, or other queries regarding your stay, you can message the property directly.
You can also make changes to your booking using the buttons below.

翻訳お願い致します。

435 :
>>434
お客様のリクエストに関し、すぐ実行できる助言を差し上げたいと思います。

支払い関連の質問、または宿泊に関する他の問い合わせについては、物件に直接メールしていただけます。
以下のボタンを使用して予約を変更することもできます。

436 :
>>432
おー 原文どうもです しかも該当箇所まで これはめちゃめちゃ参考になりました 

自分が調べた辞書にも載ってなかったです solidの不加算名詞用法
で 原文を参照しところ こっちのほうがいい解釈かもしれない

And they’ve made their footing good
and
crippled the greatest power in the world.
They’ve walked over us. The death of that one at Weybridge was an accident.
And these are only pioneers. They kept on coming.

ちなみに手元のスクリプトはこうなってる
Man Yeah. They got themselves in solid. They wrecked~

get themselves in+solid かも
つまりinは副詞のin
こうとるとsolidは形容詞で問題ない 

(参考)
形容詞の副詞的用法 
叙述用法で補語になる場合
完全自動詞の後に置かれる場合
The sun shines bright. (太陽は明るく輝く)
*brightly の意のbright はshine や burn と用いるのがふつうで,
副詞として扱っている辞書もある。詩的な表現ともされるが,話し言葉でも使う。
(ネットにあった ロイヤル英文法にのってるらしい 問題の文は自動詞の後ではありませんが)

上記の解釈に基づけば やつらの立場はびくともしない って訳は改めないといけないですねぇ
地球にしっかりとした足がかりを築いた後で アメリカを倒すわけですから ここは
地球の着実な侵入を果たしたってところでしょうか

437 :
>>436
その可能性も考えた
ちなみにsolidには副詞用法も認められていて(ランダムハウスなど)
 2.(話)完全に,十分に;しっかりと
これを使って「奴らは完全に入り込んだ」と訳したくなるところなんだけど
そうすると今度はget themselves inの形がしっくり来ない(get ... inという成句で、何かをやりとげたという意味や、誰かを呼んだという意味はあるけど、themselvesがうまくはまらない)
get inだとしてthemselvesは強調のため挿入されたとしても、get inはごくふつうに「到着する」程度。ここでget inでは軽すぎるような…
しかしこんな一見簡単な文だけど難しくておもしろいね

438 :
>>433
ちなみに そのサイトでアリスと 地球の静止する日見たんだけど
後者の映画で訳がへんなとこが少しあったかも...

まあウェルズの宇宙戦争に比べるとずっと聞きやすくて使用語彙もやさしかったですね
まあ少々ばれますが 地球の静止する日は 派手なドンパチはなく モンスター的なやつにもまったく恐怖を感じませんでしたが...

439 :
>>437
なるほど 副詞の解釈もあるかもしれない
get O in は成句 で 自分達自身を (地球の)中へ 持ち込んだ、入れたってとったけど ちがうのかな
get the washing in 洗濯物を(家の)中へ取り込む

まあ いずれにせよおそらくこんなかんじで解釈できるんでしょうね

440 :
I begrudgingly accept hot dogs as sandwiches,for the sole reason that it's impossible to come up with a definition for sandwich that doesn't either
1. exclude another type of sandwich or 2. exclude hot dogs simply because they are hot dogs or 3. includes something that is very much not a sandwich

翻訳お願いします。

441 :
I scraped my hands across the floor where she was left dying

お願いします

442 :
https://www.instagram.com/p/BgwYm1SAIwV/

最初にインタビュー受けてる逆モヒカンのホーク・ウォリアーが何て言ってるかわかりますか?

443 :
>>442
I don't know what they thoughts are exactly, except I know the four of us, are gonna take the four of them, whatever takes, Jean.
に聞こえる

444 :
>>440
短くまとめると「サンドイッチを厳密に定義できないので、ホットドッグもサンドイッチだと認めざるをえない」

445 :
>>443
what their thoughtsが正しいか

446 :
>>442
こんな感じに聞こえた
I don't know what my thoughts are exactly, except I know the four of us.
I'm gonna take the four of them, whatever it takes, Gene.

447 :
>>444
回答ありがとうございます。

ただどうして「厳密に定義できない」という訳になるのか分かりません。
最初は「1や2や3ではないサンドイッチの定義を考え出すことは難しい」だと思ったのですがそれだとおかしいですよね?
この点について教えていただけると嬉しいです。

448 :
>>441
彼女の死体が横たわる床で俺は自分の手をおしゃかにした
いい感じの日本語にするのは難しいね
詳しい状況書けばもっと正確で良い訳を誰かが書いてくれるかも

449 :
>>448
訂正
死にゆく彼女

450 :
>>448
訂正
死にゆく彼女の居る床で

451 :
>>435
翻訳有り難う御座いました。凄く助かりました。

452 :
お願いします
So I always thought Nick was the better vocalist. I was proven wrong? Since when did Joe go all growly and belting? Major vocal talent!

453 :
>>440
ホットドッグもサンドイッチであると私は渋々受け入れるはするが、その理由はひとえに、サンドイッチの以下のような定義を考案するのが不可能だからだ。
すなわち、(1)他の種類のサンドイッチを除外することも、(2)ホットドッグがホットドッグだというだけでこれを除外することも、(3)およそサンドイッチでないものを含むこともない定義。

includesだけ三単元のsがついているのと、orの入れ方からnot eitherの範囲はこの前で切れてるかもとも思ったけど、ナンバリングで示してある並列を優先した。もし切れてるとすると
(1)他の種類のサンドイッチを除外することも、(2)ホットドッグがホットドッグだというだけでこれを除外することもなく、(3)およそサンドイッチでないものを含む定義。

454 :
>>441
私は死んでいる彼女が放置されていた床じゅうを両手でこそいだ

455 :
>>452
私はずっとニックのほうがボーカルがうまいと思っていた。それが間違いだったと証明されたかな?ジョーはいつからあんなうなり声や高らかな歌声を出すようになったの。ボーカルの大きな才能!

456 :
another excellent photo

インスタの投稿画像に貰ったコメントです
普通に訳すともう一枚のいい写真、的な意味ですが、一つの投稿に単独で貰ったコメントなので「もう一枚」の解釈に悩みます
これだけで完結する褒め言葉だったりしますか?
それとももっといい写真頂戴と促されているのでしょうか?

457 :
>>453
回答ありがとうございます。

「ホットドッグがホットドッグだというだけでこれを除外することはない」=「ホットドッグもサンドイッチである」ということですよね?
ただ前者は「このような定義を考案するのが不可能」としながら後者は「受け入れる」というのは矛盾してしまいませんか?
自分の解釈がおかしいのか、それとも元の英文が間違っているのでしょうか?

458 :
>>457
>>453について、後者の場合、本来ならnot either (1) or (2), AND (3)であるべき
つまり、orがandの間違いであるか、includesがincludeの間違いであるかのどちらか。
後者だとすると、
(1)異なるタイプのサンドイッチを除外しない
かつ
(2)ホットドッグを決め打ちで除外しない
かつ
(3)およそサンドイッチらしからぬものを含む

2枚のパンで挟んだサンドイッチ以外にも、ホットサンドのような、見た目のまったく異なるもの(another type…(1))もあるわけで
そういうものも含める以上(3)、ホットドッグだけを除外する理由がない(名前で決め打ちするんじゃなければ…(2))…ということを言っているんじゃ?

459 :
>>458
回答ありがとうございます。
ただ、(1)〜(3)は考案するのが「不可能」な定義として挙げられているんですよね?
考案が「可能」なのであれば理解できるんですが、「不可能」なのであれば矛盾してしまいませんか?

460 :
>>459
現実にそういうものを考案することが不可能な以上、ホットドッグだけ除外するのは無理だと言っているのでは
細かいことだけど、「(1)〜(3)は考案するのが「不可能」」と言っているのではなく
(1)〜(3)をすべて同時に満たすものを考案するのが不可能と言っている

461 :
>>459
>>453ではなく>>458のほうの修正版の解釈でよろしく

462 :
>>460
「either A or B」は「どちらか一方」という意味で、
「同時に満たす」という意味合いでは使われないと認識していたのですが間違いでしょうか?
辞書でもそのような用法は見たことがないのですが、もし出典等あれば教えてもらえると嬉しいです。

463 :
>>462
https://dictionary.goo.ne.jp/word/en/either/#ej-27188
1b
〔否定文で〕どちらの…も(ない)(◆両方が否定される)

https://ejje.weblio.jp/content/either
b
[否定文で] (二者のうちの)どちらの…も.
I don't know either boy. どちらの少年も知らない

464 :
>>463
とても助かりました。
長々とお付き合いいただきありがとうございました。

465 :
I simply couldn’t leave your website prior to suggesting that
I actually loved the usual information an individual supply
on your guests? Is going to be again often in order to investigate
cross-check new posts https://remote.com/griffin-jim

(人∀・)タノム !

466 :
>>455
ありがとうございます

467 :
お願いします
Sorry to bring in camp rock, but Joe was picked for his vocal abilities there's no way in hell nick
would have pulled off those songs Joe sang with Demi. Clearly you haven't been listening to him singing his vocals are out of this world crazy strong and easy to listen to. Nick not so much.

468 :
>>467
キャンプロックをもちだしてわるいんだけど、ジョーが選ばれたのは
そのボーカル能力なんだよ。 なんたって、ニックがジョーがデミと歌った
数々の歌をうまくこなせるなんてことはありえない。
ジョーが歌ってるのを聴いてなかったのが明らかだね。
あのボーカルはこの世のものならずぶっとぶくらい強くて聴き心地も良い。
ニックはそこまでじゃない。

469 :
https://i.imgur.com/Siz84f8.jpg
https://i.imgur.com/81I7MeM.jpg

訳お願いします

470 :
>>456
これもまた素晴らしい写真
another excellent photo

>普通に訳すともう一枚のいい写真、的な意味ですが、一つの投稿に単独で貰ったコメントなので「もう一枚」の解釈に悩みます
他のインスタアカウントの写真も見てるので、このアカウント(あなたのアカウント)でも
良い写真見つけた、というような意味でしょう。

>これだけで完結する褒め言葉だったりしますか?
うん。

471 :
>>470
ありがとうございます!

472 :
The annual labour of every nation is the fund which originally supplies it with all the necessaries and conveniencies of life which it annually consumes,
and which consist always either in the immediate produce of that labour, or in what is purchased with that produce from other nations.

According therefore, as this produce, or what is purchased with it,
bears a greater or smaller proportion to the number of those who are to consume it,
the nation will be better or worse supplied with all the necessaries and conveniencies
for which it has occasion.

根井訳
https://webfrance.hakusuisha.co.jp/posts/199
山形訳
https://cruel.org/books/smith01/smith.pdf


, and which consist always either in the immediate produce of that labour, or in what is purchased with that produce from other nations.
このwhich の先行詞 既訳だとall the necessaries and conveniencies of lifeにとってるけど the fund じゃだめ?
とりあえず既存の訳が正しいとしていろいろ考えてみたんだが絶対ただしいとは納得できなかった

あと山形訳でfor which it has occasionの先行詞がconvenienciesになってるんだけど
all the necessaries and conveniencies じゃだめなん?

473 :
問題文の出典は国富論序文の冒頭です

474 :
While there are many ways prisons can do better at rehabilitating criminals,

訳) 刑務所が犯罪者を更生させるよりよい方法はたくさんあるが、


上記の英文がどうしてこの訳になるのか教えて欲しいです
特に prisons can do better の辺りがよくわかりません
よろしくお願いします

475 :
>>472
1 consist に s がないということは複数形だということ

2 have occasion for に
「ときどき」みたいな含意はないと思う
スミスは書中でこのフレーズを何度も使ってるので
そこから推測してみたらいい

476 :
you ever do that to me again


これneverじゃなくても
「俺に二度とこんな事すんな」って意味になるの?

477 :
ならん

478 :
>>475
山形15ページ
自分自身の消費できる範囲を超えた、自分の労働の産物を、
他人の労働の産物で必要なものの余剰部分と交換できない
からだ。

スミス
for want of the power to exchange all that surplus part
of the produce of his own labour which is over and above
his own comsumption, for such parts of the produce of
other men's labour as he [has occasion for].

おいおい「ときどき」はどこ行った?

479 :
>>474
自己解決しました
ありがとうございました

480 :
>>479
より良いがわかったか

481 :
>>480
結局そこの部分の日本語の意味の取り方を勘違いしていたみたいです

A) 刑務所が犯罪者を更生させるより、よい方法

B) 刑務所が犯罪者を更生させる、よりよい方法


最初次に続く文の意味合いからAで意味を取っていたんですが
Bの意味ならまあなんとか理解できるかなと
お騒がせしました

482 :
>>475

あざっす すっきりしました

これはまだわからん…
occasion を必要(for)ととるなら 国が必要とするすべての必要品と利便でいい気がする

>all the necessaries and conveniences of life which it annually consumes
>all the necessaries and conveniences for which it has occasion.

483 :
>>482
その解釈でいいと思う

>>478に書いたとおり
山形の訳には一貫性がない

484 :
https://i.imgur.com/3g2RzO0.jpg

これなんすけど。なんか直訳だと上手く理解出来ない。
>>476

485 :
>>484
これは命令文で、"命令文 AND 〜"で「もしそんなことをしたら〜するぞ」
YOUは主語じゃく呼びかけ。EVERは強調の副詞
「貴様、もしまた俺にあんなことをしてみろ、窓から放り投げてやるぞ」

486 :
>>485
ありがとう、超わかりやすかった

487 :
>>483
おk

488 :
A wonderful creature, but too much of a born actress to take quite seriously.

行方訳
すばらしい人だけど、生まれついての女優の才能がありすぎて、あまり物事を真面目に考え
ることができないようね

これあってる?美女グラディスのこと書いてて、以下の英文に出て来る文の訳なんだけど

このセリフは美術批評家ベレンソンの奥さんのグラディス批評で、
so changeful in a hundred moodsとかactressもあるし、takeが他動詞とすると
直訳で(グラディス)を真に受けられないみたいにならん?

英語青年って雑誌の英文解釈用に原文少々変更されてる気がする

At 16, with a beauty made disturbing by her blazing blue eyes, she was already a siren.
Her conversation shone, her profile was just short of perfection. She had many admirers
during her 25-year campaign to marry Marlborough when he and Consuelo were
divorced. The art critic Bernard Berenson was mesmerized by the teen-age Gladys, and
his wife, though jealous, felt the same.
"She is radiant and sphinxlike,” she wrote, “enchanting, but tiring. A wonderful creature,
but too much of a born actress to take quite seriously. But so beautiful, so graceful, so
changeful in a hundred moods, so brilliant that it is enough to turn anyone's head."

Eve Auchincloss, Time

489 :
>>488
自己解決しました 最初に拙訳のせてたらしい 後に次の訳があった

行方訳
驚嘆すべき人だけど、生まれついての演技者という面がありすぎるから、
どこまで本気に受け取ってよいのか分からないわね

490 :
>>456 >>471

ごめん。この人はたぶんあなたの写真をいつも、あるいは時々見ている人だろうから、
今回は1枚しかアップしていなくても、「今回もまた素晴らしい写真」という意味で書いたのだろう。
つまり、別のアカウントの写真との比較ではないだろう。

491 :
---but it's also in our nature to rebuild for tomorrow, as strong as we can.

ノートルダム大聖堂火事についてのオバマのコメントの一部なんだけど、as strong as we can の意味がピンと来ない
誰か解説して

492 :
>>491

strongが副詞で使われることもある。該当箇所は意味的には、力の限り、とかそういう感じ。

アメフトの記事から
“We got one more left, so the job’s not done, so we’re going to try to finish out as strong as we can this week and try to make history.”

493 :
>>491
まずrebuildは通常他動詞。ここでは目的語がなくても意味が通るものとして、標語っぽく自動詞として使われているが
文脈からすれば何か失われたものが入るところ
再建して、その結果その何かをどんなふうにするのか、を形容詞で示している
例・I painted the wall yellow.
地震などで家が壊れたら、ふつう、前より頑丈に作ろうとするのが人情
「できるかぎり頑強に再建する」

494 :
MARK CERNY WOULD like to get one thing out of the way right now:
The videogame console that Sony has spent the past four years building is no mere upgrade.

You’d have good reason for thinking otherwise.

お願いします

495 :
rebuildを確認してみたら、自動詞もあるようだが。

https://www.merriam-webster.com/dictionary/rebuild

intransitive verb
: to build again
planned to rebuild after the fire

例文も火事関連

496 :
>>492
なるけど、副詞ね 
ただそうだとしても前文とは、ややつながりが悪い気がするけど、ツイッターだからそこまで厳密には書いてないか

497 :
だれかこのNYTの文を訳してください。特に、were の個所を注意して

In January, just a few miles from were I met the angry local man, a car bomb exploded in the center of Derry.
No one was injured, luckily, but the entire island was unnerved.

498 :
>>495
指摘ありがとう、勉強になった
ただこの場合でもbuild againの中にその対象が含まれていると思うので、>>493は通用するはず

499 :
>>497
whereのタイポでしょ

500 :
訳はどうあれ、自分としてはas strong as のstrongは副詞だと思うんだが。
あと、つながりがそんなに悪いとは思わないけどな。

but it's also in our nature to rebuild for tomorrow, as strong as we can.

だが、未来の為にrebuildすることもin our nature なのだ。できるだけ強固に。
だが、未来の為にrebuildすることもin our nature なのだ。できるだけ力強く。
だが、未来の為にrebuildすることもin our nature なのだ。力の限り。

501 :
>>498
目的語があれば、strongは補語、よって形容詞と確定だろうが、このツイートの場合は
どうなのかね。よくわかんない。

502 :
>>500
なぜつながりが悪いと思うかというと、strongにrebuildするというのは意味的にしっくり来ないから。歴史的な寺院をもっと強固に再建築しようというのは、奇妙に思える。strongはweの心の状態を表していると解するのが自然な解釈だと思えてしまう

503 :
>>494
マーク・サーニーはいま、一つのことを退けようとしている。
ソニーが過去4年間開発に費やしてきたゲーム機は、(既存機種の)単なるアップグレードではない。

あなたがはたしてそうかなと考えるのももっともだ。

504 :
まず言っておきたいことがある

505 :
>>502
ああ、言いたいことはわからないでもない。

火事なんかに屈することなく立派に再建する、的なニュアンスなのかもしれないね。

なんか引っ掻き回してごめん。

506 :
>>501
"LIVE STRONG"といった例もあるし、ふつうに考えたら副詞か…って気がしてきた

507 :
>>505
文法的に無理な解釈を承知でいうとニュアンス的には (we can be) as strong as we can と捉えるとしっくり来る

508 :
しつこくてスマン
何でしっくり来ないか改めて考えるに
it's in our nature~ と来た時点で、その次に述べられることは普遍性があることのはず。to rebuild for tomorrow まではいい。しかし、そこに as strong as we can というような限定が入ると、普遍性が消えてつながりが悪いんだ

509 :
We will 'rebuild for tomorrow, as strong as we can (rebuild strong)'
じゃないの

510 :
strong 副詞 力強く 強く 強烈に 猛烈に 激しく 勢いよく
To maximize the impact of any written document-electronic or traditional-you must start strong.
Eメールであれ 伝統的なメールであれ 文書のインパクトを最大限にするには力強く書き始めねばなりません
The tide is running strong. 潮の流れは激しい

511 :
原文これか
Notre Dame is one of the world’s great treasures,
and we’re thinking of the people of France in your time of grief.
It’s in our nature to mourn when we see history lost –
but it’s also in our nature to rebuild for tomorrow, as strong as we can.

どっちにしろおなじ気がする
but it’s also in our nature [to rebuild for tomorrow, as strong as we can (rebuild strong)].

512 :
どうでもいいけどオバマってフォロワー1億人もいてワロタ

513 :
>>503
ちょっと酷すぎる訳だよ

514 :
>>503

get one thing out of the way right now:
これ、ジャマなものをかたすといった意味だと思うけど

You’d have good reason for thinking otherwise.

こいつは、"じゃないと思うんなら、シッカリした理由がないとね"

といった意味だと思う

515 :
長文ですが、こちらの翻訳をお願いします。医学用語なので難しいです。

Microsomal cytochrome P450 enzymes (CYPs) are membrane-attached enzymes
that play indispensable roles in biotransformations of numerous endogenous and
exogenous compounds. Although recent progress in experiments and simulations
has allowed many important features of CYP-membrane interactions to be deciphered,
many other aspects remain underexplored. Using microsecond-long molecular dynamics
simulations, we analyzed interaction of CYP3A4 with bilayers composed of lipids differing
in their polar head groups, i.e., phosphatidylcholine, phosphatidylethanolamine, phosphatidylserine,
and phosphatidylglycerol. In the negatively charged lipids, CYP3A4 was immersed more deeply
and was more inclined toward the membrane because of favorable electrostatic and hydrogen
bonding interactions between the CYP catalytic domain and lipid polar head groups.
We showed that electrostatics significantly contributes to positioning and orientation of CYP
on the membrane and might contribute to the experimentally observed preferences of individual
CYP isoforms to distribute in (dis)ordered membrane microdomains.

516 :
長文の場合は主旨や概要の質問くらいにして翻訳を頼むなら有料のとこに依頼するとか配慮はないんだろうか…

517 :
For English users who might be confused, Google translate offers a better translation for the "Location to be added" line: "This mod adds the following locations". In other words, it's already functional as is. =)
PCゲームのModを作成して英語で貰ったコメントなんですが
「グーグル翻訳で英語説明を追加するとかえって分かり辛いからもっと短くて良い」
って感じでしょうか

518 :
>>517
混乱しているかもしれない英語話者のユーザーには、「追加すべき場所」の行のところは、グーグル翻訳がもっとわかりやすい訳を出してくれる。つまり「このMODは以下の場所を追加する」。つまり、このMODはこのままでもう使える。(ニッコリ)

ユーザーが場所を自分で追加しないといけないのかなーって誤解が生じかねないということかと

519 :
>>503
マーク・サーニーはいま、一つの問題にケリをつけようとしている。
まで訳せばよかった

520 :
>>506
そうだね

>>508
なるほど、するどいね。

521 :
And what the waur am I?
以下のように訳せるらしい
なんにも困ることありゃしない

comin' through the ryeって詩の一節なんだけど 文法的にどう説明できるかわかりません
waur=worse
theが副詞で反語っぽいけど
疑問符があるのでwhatは 疑問代名詞だと思うんだけど それだけ悪くどんな人になるっていうの?どうという人にもならないわ=べつにどうってこともないわ
みたいな解釈でいいんですかね?

522 :
>>521
疑問副詞じゃないかな
What does it matter? みたいな。
ジ大では(通例皮肉的)とされている。

waurについては
"waur: worse off. "との注を与えているところがあった
https://rpo.library.utoronto.ca/poems/comin-thro-rye

https://www.ldoceonline.com/dictionary/worse-off
2 in a worse situation

theは指示副詞として、what→the→worse offのように修飾している
「それでどれだけ状況が悪くなってるっていうのさ。」

523 :
レスありがとうございます
出典だしてなかったですね (Comin thro' the Rye)

waurはworse off.ととれるんですか これは勉強になりました
Longmanだと2のin a worse situationっていみでしょうね

疑問副詞もかんがえたんだけど
内容的に(意味的に) what(どれほど) ってのと the (それほど)って矛盾する感じがしたんですよね
the はコンテクストからして おそらく(みんなほかの女子にはいるのに自分には)恋人がいない状況だと特定されてて
whatとどう合うのだろうと...

で waur がworse off の in a worse situation として
And what the waur am I? をAnd what am I in the worse situation? にできたとして解釈すると
やっぱり
それほど悪い状況だからといって何になるというの、何にもならないわ って感じになる気がします
まあ、あたらずとも遠からずみたいなとこでしょうか

524 :
>comin' through the ryeって詩の一節なんだけど 
出典だしてたな(´・ω・`)

525 :
>>523
so much the worse
のような例ではthe worseを副詞が修飾しているよね
https://ejje.weblio.jp/content/so+much+the+worse
https://dictionary.goo.ne.jp/word/en/worse/
・ So much the worse because you're a member of the club.
・ 君がそのクラブのメンバーなのでますます具合が悪い
このso muchを疑問詞にしたら、問題の文のようになるはず

526 :
このtheはおそらく次のような説明がのってるtheだとおもいます

ジーニアス
[比較級の前で]それだけ ますます かえって
それだけが呼応する理由を表わす語句はしばしばfor, since, becauseなどで導かれる

I like him all the better for his frankness.
彼はさっぱりした性格だから一層好きだ。
If we start now, we will be back the sooner.
今出発すれば それだけはやく帰れるだろう
The danger seems to make surfing the more exciting.
サーフィンは危険だからこそいっそう楽しいようである

soは 程度 それほど そんなに

so much the worse because you’re a member of the club. の文で
the はbecause以下と呼応してて、どの程度か示されてるように思います

soにしろtheにしろ程度は特定されている(程度が分かっている)ので、
これをもとにwhatをつかって程度を尋ねる疑問文(程度がわからない)をつくろうとした場合、
the worseとは両立するのかなとおもいました。

527 :
お願いします
Back then joe was the main star now it’s nick.. I feel bad for kevin

528 :
あの頃はジョーが花形だったってのに、今はニックだ
……ケビンが残念でならねぇ

529 :
ありがとうございます

530 :
>>526
そのtheですよ、もちろん

その理屈でいうと「so much the worseの場合はsoとtheの両方で程度を示しているので冗長だ」ということになりそうなもんだけど、そこは気にならないんだ?

"what the worse 〜"でtheもwhatもworseの程度を説明していると思っているのかな
実際は>>522で書いたとおりwhat→the→worseのように修飾している
別の表し方をすれば(what (the (worse)))

つまりthe worseにも程度がある。ある理由が原因で、それだけ〜〜になる場合、
完全にその可能性を実現していれば all the worse
全く実現していなければ none the worse
ほんの少しであれば a bit the worse なんで表現も可能だろう
("I don’t like your gown a bit the better for being French.")
https://www.econlib.org/library/Topics/College/balanceoftradeandbalanceofpayments.html

531 :
>>530
まあ不定(what)と定(the)の矛盾に比べれば気にならないですね
冗長がいけないというのならall the+比較級もallもだめでしょう

I like him none the worse for his faults. 欠点があるがやはり彼が好きだ.
He is all the better [worse] for a change of air.
転地をしたのでそれだけよくなった[かえって悪くなった]
You'll be all the better for a rest. ひと休みすれば(それだけ)気分もよくなりますよ.
He is none the worse for the horrible results of his marriage to Andrea.
アンドレアとの結婚はひどい結末を迎えたが、彼は少しも変わらない。

ジーニアス
all the+比較級 かえって ますます theはそれだけ(よけいに)(by so much)
新英和中
all [the+比較級の前に用いて] それだけ,ますます,かえって.

Noneもこれで意味的矛盾が生じることはないので気にならないですね
意味に矛盾がないのであれば別にこの位置に程度の副詞がきてもなんともおもわないですね

532 :
Clearly, some Trump advisors restrained his worst impulses
(and paid a price).
Attempted murder is a crime, so is attempted robbery
and attempted arson.
What about attempted obstruction?

翻訳お願いします

533 :
>>532

明らかにトランプのアドバイザーたちの何人かは彼(トランプ)の
最悪の衝動を抑制した(そしてその代償を払った=トランプに
首にされた、政権を去った)
殺人未遂は犯罪だ。強盗未遂も放火未遂も犯罪だ。
司法妨害未遂はどうなのだ?

534 :
>>533

ありがとうございます

535 :
和文を英訳して作ったのですが訳でおかしなところがあったら教えてください。

●日本人はもっと「いい加減」に生きた方が幸せになれる
Japanese can be more happy if they live more and more flaky.
●海外旅行から日本に帰ってきてまず思う事は、日本の清潔さと静寂です。
When I came back to Japan from overseas travel, it comes to me think of Japan first,
there is the cleanliness and quietness in Japan.
●そして、もう一つ感じるのは、あちこちに見られる「過剰なサービス」です。
And I got the feeling what there are providing service too much everywhere,
such as public facility, restaurant and convenience store.
●例えば、東京のメトロに乗っていると、ダイヤの時刻にわずか数分遅れただけで、
車内アナウンスで車掌さんが謝罪をしています。
In case of public facility, train service, while you are riding on a Metro of Tokyo, the anouncement
of the conductor is always apologizing for only few minutes behind the service planning diagram.
●そもそも遅れた原因は運転の問題ではなく、乗客の乗り降りに時間がかかり過ぎたり、
事故や故障です。しかも、それが数分の違い。なぜ、毎回謝る必要があるのか私には理解できません。
That means the cause of delay is not only a driving problem incidentally, but also it takes too long for
passengers to get on and off, accidents and bleak-down happened, and that is just couple pf minutes
behind schedule. This is what I do not understand why they have to sorry every time.

536 :
My Father will be in Japan in May. He would like to visit everyone. He is going there for my Mother’s 3rd year of passing. I will send itinerary can you give to your Grandmother (Sizuko) and your Uncle (Masato) please.

先生、これお願いします。

537 :
このキチガイ連呼の工作員は、証拠があがってバレてるのだから、雇い主はこいつを即刻解雇しないと評判落とす。

ワッチョイ f623-EKn6 スップ= Sdb2-cca7=HKW 0H71-W+7L=アウアウエー Sada-Xm7k

上記の正体

英語板は一人の工作員の自演レスで水増しされてる
https://lavender.2ch.sc/test/read.cgi/english/1555575841/

キチガイ連呼は工作員の証明。

538 :
>>536
私の父は5月に日本に行きます。父は皆さんを訪ねたいです。父は私の母の三回忌の為に行きます。旅程表を送りますので貴方の祖母(Sizuko)と叔父(Masato)に渡して下さい。

私の日本語が上手くないけど許してね
それより、プライベートな事を名前付きでこんな所に書き込んでいいの?

539 :
>>538
うわ、すげぇ。
コピペの事情でID変わるけど、
続き送りますね。
すいませんが、お願いします。

540 :
Has your grandmother been sick? My Dad will be there in May. Please tell everyone I said hello. Hugs!

お願いします。先生

541 :
三年英太郎が英語板の自演業者だったってマジかよ

542 :
>>540
おばあちゃんは元気?父が5月にそちらに行くから、みんなによろしく伝えてね。ハグ!

意訳だけど

543 :
>>542
マジ?直訳太郎なんで最初の文章は
「ババァはずっと病気?」ぐらいの勢いで読んでたわ。

544 :
Trump promised to fight for American workers.
He lied.
Americans deserve a president who will.

翻訳お願いします

545 :
トランプはアメリカ人労働者のために闘うと約束した。
彼は嘘をついた。
アメリカ人は本当に闘う大統領を選ぶべき(時だ)

546 :
>>545

ありがとうございます

547 :
My Father says hello. He will be there May 3rd and would like to visit you and your family.

He will be in Fukuoka on May 3rd and if you would like to see him he can meet you. He’s celebrating my Mother’s 3 years of going to Heaven.

お願いします、先生

548 :
Those boots though��

SNSにレスラーが2人で組み合っている画像をアップしました
それに寄せられたコメントです
確かに2人はブーツを履いていて、うち一人のブーツは文字がデザインされていてとても派手です
ビックリの顔文字も付いているので「ブーツもなんか凄いね」みたいなことを言われてるのかなと思ったのですがどう解釈すれば良いでしょう?

549 :
>>547
父がよろしくと言っています。父は5月3日にそちらに着き、あなたとご家族のところにお伺いしたいと思っています。

父は5月3日に福岡に着きますが、もしあなたが父に会いたければ、父はあなたに会えます。父は母が天に身罷って3周年の弔いを行う予定です。

550 :
>>548
にしてもそのブーツって
ぐらいかな?顔文字は化けてて見えない

551 :
>>550
顔文字ごめんなさい
円に黒点3つのビックリ顔でした

ニュアンスなんとなくわかりました
ありがとうございます!

552 :
「(スマホが)いつ切れてもおかしくないです」

英訳お願いします、先生
(電波じゃなくて電源の方です)

553 :
>>552
My phone is about to die.かな?
スレチだけど

554 :
obviously nick he’s been famous since they broke up the rest were kinda irrelevant nick has been in huge movies
お願いします!

555 :
>>553
ありがとうございました!
ここ、英文→和文でしたね。
スミマセン。

556 :
GoT: As a long-time storyteller,
I'm in awe of how perfectly the minds behind this show brought
all the major characters together at Winterfell.
They made it look easy. Constant Readers, it is not.

翻訳お願いします

557 :
>>556
『ゲーム・オブ・スローンズ』について:長い間物語を書いてきた人間として、
この番組の制作者たちが「ウィンターフェール」の回で、主要な登場人物全員を
集結させた手腕の見事さに畏敬の念を抱いている。
彼らはそれがかんたんに見えるようにした。わが愛読者諸君、実はちがうのですよ。

558 :
>>554
あきらかに彼らが解散して以来ずっとニックは有名だった残りの方はなんていうか空気だったニックはずっと大作映画に出てた

559 :
>>557

ありがとうございます

560 :
コンマもピリオドも大文字小文字の区別もない文をよく読めるなぁぁ

561 :
ニックがどうとかずっと書き込んでる奴はなんの本読んでるんだw

562 :
>>558
ありがとうございます

563 :
As the waterwheel turned, they were lifted up out of the river and on to a platform at the top.

精読でお願いします (*^^*)

564 :
>>563
水車が回転し、彼らは川から、水車のてっぺんのところにあるプラットホームまで引き上げられた。

565 :
>>564
ありがとうございます。
この場合の
asはどういう用法だと考えられますか?

566 :
>>565
asの根本的な意義は同時性だと思うんだよね
理由としてとらえてもいいと思うけど、あえてやんわりと繋げる接続詞がasだと思ってる
主節のほうに比較級があれば「〜につれて」と訳せ、経過的な描写がしてあるので実際ここではそう訳したくもある

567 :
>>566
丁寧にありがとうございます。
前後の文脈的に「〜に連れて」っぽいのですが「〜に連れて」と読んでも文法的な矛盾はないでしょうか?

あと、
詳しく聞きたいことが多いので今後はこっちのスレで質問させてもらう事にしました。
http://itest.2ch.sc/test/read.cgi/english/1522564281/836
今回はマルチになってしまってスミマセン

568 :
よろしくお願いします。

It was a dramatic comeback years in the making for one of the world's most famous athletes.

世界で最も有名なスポーツ選手の1人が、長い年月を経て劇的な復活を果たしました。

in the making for の辺りを、もう少し直訳風にすると、どんな感じになりますか?
お手本の訳が全くピンと来ないです。

569 :
>>568
It was a dramatic comeback (years in the making)
for one of the world's most famous athletes.

570 :
・・・分からないです
もうちょっと詳しくお願いします

571 :
in the making 「進行中」の意味の形容詞句
years → in the making 「何年も進行中」
a dramatic comeback ← years in the making 「何年もかけて進行したドラマチックな復活劇」

572 :
ありがとうございます!
何年もかかっ「た」っていう、過去形になるのがちょっとスッキリしないんですが、
その辺は臨機応変に訳す感じなんでしょうか

573 :
カムバックという出来事を
「作るのに・製作に」in the making
何年も・複数年years
かかった・あったwas
てことかね・・・

574 :
>>572
素直に読んだら「何年もかけて準備中の劇的な復活劇」だね
文脈を想像して、この文が指している時点でa dramatic comebackは実際に達成済みのことと想像して訳したけど
まだ未完なら進行中の訳のほうがいいと思う
どういうシーンで言われた文なのかとか、itの中身がわかれば、どちらが適切かはっきりできると思う

575 :
タイガーウッズがこの前の試合で優勝して、
長い間の不調からカムバックしたっていうニュースだったので、
過去形が適していると思います。

ほんとにありがとうございました!
とても納得できました。

576 :
>>575
これって口語?

577 :
Donld Trump has a ring around his ass from sitting on the pity pot.
Are people really going to vote again for this crybaby?

翻訳お願いします

578 :
i wouldnt, when you push your tongue, just push it to the top of your teeth on the gum instead.

お願いします

579 :
She has beautiful cubs
動物園の動物について話してる時の文です
よろしくお願いします

580 :
かわいい子どもがいるよ

581 :
cubは主に肉食獣の子ども

582 :
>>580
ありがとうございます
なんとなく文章が繋がった気がします
助かりました

583 :
>>581
肉食獣というか猛獣でも同じでしょうか?

584 :
>>583
オオカミやクマやパンダなど

585 :
>>584
ありがとうございます

586 :
イラストを見た方から以下のメッセージをいただいたのですがいまいち意味を掴みかねています
翻訳をお願いします

Hello, I am curious, do you make commissions?

587 :
>>586
興味あります。絵を依頼してもいいですか?

588 :
>>587
素早い翻訳ありがとうございます!
助かりました

589 :
Bank that pays its CEO
$31 million and received a $12 billion bailout
after crashing our economy tells
poor people to stop being so irresponsible with their money.

翻訳お願いします

590 :
>>589
CEOに年に3100万ドルの報酬を支払い、我が国の経済をぶち壊した挙句120億ドルの公的資金で救済して
もらった銀行が貧しい人々に対し、自分のお金に無頓着すぎるのをやめなさいと言い聞かせている。

591 :
要はお前が言うなってこと。

592 :
意味がとれるようでいて自分には日本語に訳しにくい文です
どなたかお願いします

A gauche remark is tactless; a gauche offer of sympathy is so bumbling

as to be embarrassing; gaucherie is an awkward, clumsy,tactless, embarrassing way

of saying things or of handling situations.

The gauche person is totally without finesse.

593 :
こちらは前置詞on、throughに引っかかっています
throughはくり抜くという感じかもしれませんが、cut onがわかりません
よろしければお願いします

Imagine a book, a complicated or massive report, or some other

elaborate document―now figuratively cut on or through it so that

you can get to its essence, the very heart of the idea contained in

it. What you have is an epitome, a condensation of the whole.

594 :
Get your asssssss over here nowwwwww

ツィッターでの海外の方からのリプなんです
自動翻訳では「いまここにあなたの評価を得る」となったのですがイマイチわからなくて
よろしくお願いします

595 :
>>594
今ここに直接来いよ

596 :
>>595
ありがとうございます!

597 :
>>590
ありがとうございます

598 :
次の英文で質問が2つあります。

When you do formulate the purpose statement,
focus on what the function computes not how it goes about it,
especially not how it goes through instances of the given data.

・自分の訳
目的文を組み立てるときは、関数がどのように処理するのかではなく、
特に与えられたデータ・インスタンスをどのように処理するのかではなく、
関数が計算するのは何なのかに焦点を合わせます。

質問1
@「not how it goes about it」のitはそれぞれ何を指しているのでしょうか?

質問2
「gooes about」と「goes through」を両方とも『処理する』と訳しましたが、
もっと最適な訳はありますか?

599 :
再度ですみませんがお願いします。
上の歯の裏に対して舌を押し付けるように指示されたときに
前歯が弱いので歯の裏は難しいという文の中での返答がこれです。
翻訳お願いします。
i wouldnt, when you push your tongue, just push it to the top of your teeth on the gum instead.

600 :
>>599
わたしはやらない(?)
舌を押し上げるときに、代わりに歯の上部の歯茎の部分に押し付けてみてよ

i wouldntの部分はそこで切れて、前文のなにかを否定していると思うんだけど
前文(英文)がないのでよくわからない
最初「私は歯の裏に押し付けないよ」かなと思ったんだけど
指示した人がそんなこと言うはずないよね

601 :
>>599
舌を歯と歯茎の間に押し付けるべし

602 :
>>598
これ、文脈 書かねえと さっぱりコメント付かねえんじゃねえの?

"perpose statement" で ぐぐってみたら、
『(関数とかの) 入力データに対する プログラムの作用 を書いたもの (コメント)』
みたいなterm っぽいが、そういう文脈か?

* Design Recipes in C
< https://hci.uni-hannover.de/files/prog1script/script.html >

> When you do formulate the purpose statement,
> focus on what the function computes not how it goes about it,
> especially not how it goes through instances of the given data.

"purpose statement" ((関数処理の) 目的文) を書く際は、
『関数が 何を計算するか (what the function computes)』に着目します。

細かい処理手順 (how it goes about it)、
特に 具体的なデータの例 について どのように処理するか
(how it goes through instances of the given data)
を説明し始めてはいけません。

603 :
韓国某社のTOEIC模試からです。
Seven of the eight letters referred to have been preserved.
「8通のうち7通の手紙が保存されていると言われている」というようなことを言いたいんでしょうが
refer toの次には名詞が来るはずなのに完了形不定詞なんて変ですよね。
完了形不定詞が名詞用法だとしてもreferの意味が通りません。
単純なミスでしょうか?

別問題で、学生が「teachingが終わったら行くよ」みたいなことを言っていますが
teachingは教員側から見た授業のはずですよね。
学生がteachingと言ったら、学生自体が授業をすることになってしまうと思います。

ご教示願えたら幸いです。

604 :
>>603
ご指摘の8通の内、7通が保存されている。って意味では?
referred to = ご指摘

605 :
>>604
なるほど、referred toは主語にかかっているんですね
スッキリしました
ありがとうございます

606 :
>>600 ありがとうございました
>>601 ありがとうございました

607 :
この日本語訳お願いします

I think he knew. If you notice, he was writing when he saw something and looked up at her while she was looking away.
He must have seen her name, looked at her to confirm, then acted like nothing happened.
It was perfect too, because she never noticed him starting at her.

608 :
>>607
俺は彼は知ってたと思うよ。彼女が遠くの方を見てたとき、彼は書いていたけど、何かに気づいて、彼女を見上げたよね。
彼は彼女の名前を見たと思うし、顔を見て確認した。そして、何もなかったかのように振る舞った。
その動きは完璧でもあった。だって彼女は彼に見られていたことを全く気づかなかったからね。

609 :
>>608
ありがとうございます

この動画の会話の翻訳もお願いします
英語の字幕がついてるんですが
https://www.youtube.com/watch?v=dNyNEO4Ef08

610 :
この会話の翻訳お願いします

A 「Kimberly Deal. Like Breeders」
B「Right」
A「Listen to The Breeders?」
B「Huh?」
A「Listen to them?」
B「Do I listen them?」
A「No」
A「They're really a cool band,actually」
B「Is he fucking with me?」

611 :
>>609
>>610
店員「Kimberly Dealですね。Breedersだっけ。」
客「ええ。」
店員「Breeders聴いたりします?」
客「えっ?」
店員「彼らの曲聴いたりしますか?」
客「私がですか?いえ。」
店員「結構いいバンドですよ。」
客「(私のことからかってるのかしら...)」

612 :
>>611
ありがとうございます

613 :
>>608
天才現る
スゲー

614 :
About as far from the Equator as you can get in Japan proper.

お願いします

615 :
自己解決
japan proper で 日本本土ですね

616 :
i truly appreciate it a lot

インスタにかっこいい画像あげたらもらったコメントなんですけど普通に「超ありがとう」みたいな意味で良いんでしょうか?
最後に青い顔で叫んでる顔文字付いてたんでちょっと不安になって
返事するとしたらyou are welcomeとかでいいのでしょうか?
なんか固い気もしますが

617 :
@ICYMI: In 1999,
Lindsey Graham argued that
"impeachment is not about punishment.
Impeachment is about cleansing the office."

A"That's the sort of video...
that make people so skeptical and so disgusted
with politics"

翻訳お願いします

618 :
>>617
弾劾すべきは罪ではない。
官庁だ。1999年
ご存知だとは思いますが。

ある種の映像により政治に懐疑的、反感を持つ様になる。
disgust = dis + gust = dis + taste
gust = レストランガスト

619 :
>>616
「この画像が見れてほんとによかった」みたいな感じかな。
何か、映像でも音楽でもいいけど、そういうものに触れて、この音楽が聞けてよかった、この音楽のよさを理解できて享受できた、と思うことがあると思う。
You are welcomeは、個人的には Thank you と対になってる印象があるので、
I'm glad you liked it. (気に入ってもらえてよかった)とかどうかな。

appreciate で thank you とほぼ同じ直接的な感謝の表現の場合もあると思うけど、
you are welcome を thank you 以外の場合に使うことが自然なのかどうか(俺は)わからないので、
そういう場合でも It's my pleasure. とか No problem. とかと言っておく方が無難かもしれない。


>>617
1. 弾劾は罰するためではない。弾劾は、組織を浄化するためだ。とLindsey Grahamは主張した。
2. このビデオは、政治に対して人々を懐疑的にし、反感を持たせるようなビデオだ。

620 :
>>619
2段落目訂正。

appreciate に対して you are welcome が自然なのかどうか(俺は)わからないので、
直接的に感謝が表現されているような場合でも、it's my pleasure. とかの返し方の方が無難なのかもしれない(し、you are welcome でも自然なのかもしれない)。

(そして今回は、直接的な感謝かどうかも不明なので、
I'm glad you liked it. あたりが無難かと。)

621 :
>>618
>>619

ありがとうございます

622 :
Of course for years some people have told me
I don't know how to end a story.
I call bullshit on that, but everyone has an opinion.

翻訳お願いします

623 :
>>619
レスが遅れてすみません!ありがとうございます
こういう私への直接的ではない「神に感謝」的なコメントもインスタでの頻出ワードな気がしてるので
返信の仕方今後も参考にさせてもらいます

624 :
>>622
驚くまでもなく、もうずっと、私は物語の終わらせ方がわかってないと、一部の人から言われ続けてきた。
ご冗談をと言いたいが、人にはそれぞれ意見ってものがあるからね。

625 :
>>624
>I call bullshit on that
はなんで「ご冗談をと言いたいが」になるの?

626 :
>>625
https://www.urbandictionary.com/define.php?term=I%20call%20bullshit
>By doing this, they are expressing their disagreement with what the person said in a humorous and yet serious way.

627 :
>>624

ありがとうございます

628 :
>>626
スッキリ。ありがとう
一種の決まり文句だったのか

629 :
「Lelouch of the Re;surrection」という映画の会見動画なんですが、
動画の女性がなんと質問してるか翻訳お願いします

Goro Taniguchi Director Spotlight at Sakuracon: Code Geass questions
https://youtu.be/DZ9fYIZLM5Y?t=65
(1:05くらいからの質問)

630 :
>>629
ルルーシュが復活するのは
プロジェクトの始めから決められていたことなのですか
それとも、制作を通じて熟考され決められたのですか

631 :
>>630
英文としては何て言ってるの?

632 :
Given this is The Lelouch of the Re;surrection, I *** *** was Lelouch's resurrection decided from the beginning of the project or was it deliberated, ah, kind of throughout the d**?

633 :
>>630>>632
おおー
超ありがとうございます!
全然わからなくて、助かりました

I *** *** was のとこカリキュラ…?とか聞こえるけど何て言ってるんだろう
the d**?のとこはdevelopmentととかなのかな…?

634 :
Given this is The Lelouch of the Re;surrection, I was kinda curious, was Lelouch's resurrection decided from the beginning of the project or was it deliberated, ah, kind of throughout the development?

635 :
>>634
was kinda curious
な、なるほどー
カリキュラム?とかかと思ったら全然違ったw
ありがとうございました!

636 :
>>634
すごいな、よく完璧に聞き取れるな

英文を見た後でも、よく見たらちょうど>>632と同じ二箇所が俺も聞き取れなくて、
10回くらい聞き直してどうやら634で完璧に合ってるっぽいとわかったわ

637 :
Last time pooping in Japan for a while at least.
お願いします

638 :
日本でうんこしてから結構経ってると思うわ

639 :
こういう文書て文法的にはネイティブから見てどうなんだろうか

640 :
This is not the Obama Administration, or the Administration of Sleepy Joe, who let China get away with “murder!”

中国を葬り去っても構わないって書いてあります?

641 :
>>640
この政権はオバマ政権でも、中国に好き勝手させた「寝ぼけのジョー」(トランプがバイデンに付けたニックネーム)の政権でもない。

642 :
Besides, may I learn if is possible to pick up the wrong parcel by next Monday(May.13) or Tuesday(May.14)? If is okay, please let us know what date is more suitable for you.

翻訳お願いします

643 :
私は次の月曜(おそらく13)に不正な荷物を取り戻して学習か?オーケーなら情報が最適なあなたの知らせる。

644 :
>>641
なるほどどうも!

645 :
>>641
これはすぐれた翻訳だ

646 :
翻訳お願いします

Why are you people hating? That woman is beautiful and done no wrong by you.
I bet if she ask any of idiots out, you would break your necks saying YES or you would not be watching the video.
While some of you trippin off her walk, have any of you fools notice that big as ring on her finger?
Is that why some of you women hating? She seem to me to be happy with her life, while you negative minded fools are miserable with yours, go get a life already.
Just keeping it 100%

647 :
これどこかおかしいですか?
意味通じます?

In Japan, it is true that dishonest and deceptive behavior are regarded as virtues in certain situations.
But isn't it the same in other countries?
For example, A white woman be asked a question "would you date an asian guy ?"
She doesn't really want to date an Asian guy, But she answers:"It's the individual who matters, not race."
This is dishonest and deceptive answer, but it comes from kindness to avoid hurting others.The same applies to japanese people

648 :
意味を書け

649 :
>>648
(dishonestでdeceptiveなのが日本の文化だ、というレスへの返答)

日本で、特定の状況下で、不正直で本心を隠すような言動が美徳と見なされているのは事実です。
しかし、他の国でも同じではないですか?
たとえば、白人女性が「アジア人男性とデートするつもりはあるか?」 と質問されたとします、
彼女は全くアジア人とデートをするつもりはありません。しかし彼女は「人種ではなく、重要なのは個人です」と答えます
これは不正直で本心を隠した答えですが、他人を傷つけないようにするための優しさからきています。日本人もこれと同じことが言えます。

650 :
>>647
とりあえず

>For example, A white woman be asked a question "would you date an asian guy ?"
>She doesn't really want to date an Asian guy, But she answers:"It's the individual who matters, not race."

For example, if a white woman is asked if she wants to have a date with an Asian guy and doesn’t really want to, she’ll answer “It’s what he is like that matters, not where he is from. “

かな

651 :
Hotel Northern Lights is an international 5-star hotel, located in the heart and true center of Helsinki.

trueがよく分からん

652 :
その文だと寧ろ何でorではなくandなのかわからん...

653 :
文字通り

654 :
文字通りだとしたら or じゃね?

655 :
>>651
普通に「ど真ん中」ってことじゃないの

656 :
>>650
ありがとうございます。

657 :
2つの名詞、heart and center が、of 〜とつながっていて、centerにtrue という形容詞が付いた形だと思うのですが、
true center でど真ん中って意味になるんですか?
true center という連語で検索してもよく分からなかったです。

658 :
>>657
heartもcenterも同じようなこと(どちらも「中心」)なので、二回繰り返して強意になってるだけだと思う
centerにはさらにtrueまでついて、より強意されている

「ノーザンライトホテルは世界的な五つ星ホテルで、ヘルシンキのど真ん中に陣取っています」
くらいの意味だろう

659 :
Nick is using a lot of falsetto because he can, not many artists are that good in this category, he is very good. He proved himself, we know their songs from past. The difference between them is actually small, both of them are good but in different ways

お願います

660 :
医薬品を抱えた男性が殺されているシーンなのですが

You spot something a surgeon should supervise shielded from simple sight on the slashed stiff.

よろしくお願いします

661 :
Bill Cohen, on Trump:
“He is basically thumbing his nose at Congress and saying,
‘I don’t recognize you.’”
Republican congresspeople have abrogated their responsibilities
to provide a necessary check on Trump’s behavior.

翻訳お願いします

662 :
>>646
黒人英語っぽいというか、文法きっちりしてなくてスラング多すぎなので正確じゃないかもしれんけど

あんたらなんでヘイトするわけ? あの人はきれいだし、あんたらに迷惑かけたわけじゃないじゃん?
もし彼女があんたらバカどもの誰かを誘ったら、あんたらは尻尾振ってオーケーするか、そうでなくてもこの動画は見てないよね?
あんたら歩いてる彼女の悪口言ってるけど、アホなあんたらにも彼女が大きい結婚指輪をしているのに気づいた奴がいるってこと?
ひょっとして、女もヘイトしてるのはそれが理由なわけ? 彼女は自分の人生に満足してて、あんたらネガ厨どもは自分の人生を呪ってるようにしか見えんわ。引きこもってんじゃねーよ、マジで。
マジレスしちまったぜ。

to ask someone out = デートに誘う
to break one's neck = 大変な努力をして何かをする
to trip someone off = 欠点をあげつらう
as ring = assring = 肛門の周りの環状組織。そこから派生して、結婚指輪を意味するスラング
go get a life = ソーシャルな関係を持てる友達を探せ

663 :
>>659
ニックはファルセットを使えるので良く使っている。このジャンルで(ファルセットが)うまいアーティストはあまりいないが、その中でニックは非常に優れている。
過去の曲をみても、彼は実力を示している。彼ら(ニックともう一人の歌手?)の間の差は実際のところ小さい。方向性は違うが二人とも優れている。

664 :
>>660
この斬死体を一見しただけはわからないところに隠されたところに、外科医の指導が要るようなものを見つけました。

医薬品であることを隠さなくてよければ、「医者にしか使えないようなヤバイい薬」とかにした方が自然かも。

665 :
>>661
トランプについてビル・コーエンは「基本的に彼は議会に対して、相手を馬鹿にした仕草をしながら『お前のことなんか知るか』と言っているようなものだ」と述べた。
共和党議員は、トランプの態度について必要なチェックを行うという自分たちの責任を放棄している。

これ、二番目の文もクオーテーションで囲まれてるんじゃないかな?

666 :
>>660
あなたは医者が扱うような物が死体に上手く隠されていたのを見つけた。

667 :
>>662
ありがとうございます

668 :
what was at first a strategy to avoid terrorist attacks and property crime has been extended into a system to monitor anti-social behavior, like graffiti and littering.

ちょっと長めの一文です 翻訳お願いします

669 :
最初はテロや窃盗を防ぐための戦略だったものが拡大され、落書きやポイ捨てのような反社会的行動の監視に使われている。

670 :
>>664
>>666
ありがとうございました!

671 :
>>669
ありがとうございます
あと一文だけ翻訳お願いします

the biggest danger is perhaps not the individual technologies, but the overall idea (that the surveillance culture is necessary and even in our best interest s)

672 :
>>665

ありがとうございます

673 :
>>671
最も危険なのはおそらく個々の技術一つ一つではなく、全体的なアイデア(監視文化が必要でありそして我々に益するという考え方自体)だ

in one's best interest 〜とって一番利益になる、一番有利、〜のため (必ずしも経済的な利益だけのことではない)

674 :
>>673
助かります ありがと

675 :
During year 2014, Cerrone established an MMA gym at his property in Edgewood, New Mexico to lower his training camp costs by hosting his training partners at the estate.
Coaches like Brandon Gibson, Jafari Varnier and Jonavin Webb are usually situated at the ranch with fighters like Lando Vannata, John Dodson and Leonard Garcia


翻訳お願いします

676 :
>>675
2014年の間にCerroneはニューメキシコ州エッジウッドの私有地(or 所有する建物)にMMAジム(総合格闘技ジム)を
設立した。設立目的はトレーニングパートナーらをそこに迎え入れる(or 泊まらせる)ことでトレーニングキャンプ
費用を削減することだった。

Brandon Gibson、Jafari Varnier、Jonavin Webbらのコーチは通常、
Lando Vannata、John Dodson、Leonard Garciaなどのファイターたちと共にトレーニング
ジムにいた。
ーーーーーーーーーーーーーーーーーーーーーーーーーーーーーーーー
ranchは普通は牧場だけど、このトレーニング施設のことをranchと呼んでいるらしい。

677 :
これおねがいします
i think it's exciting chatting with different people...
would you like to chat today?
where are you located?

678 :
>>676
ありがとうございます

679 :
>>677
色んな人(他人)とチャットをするのは楽しいことだよ
今日は話したい(話せれる)?どこに住んでる?

680 :
He turned out to be a liar when all was said and done.

彼自身が嘘吐きなのか、誰が嘘吐きかは判らないけど嘘であることを彼が判ったのか?
ちょっと混乱してるのでお願いします。

681 :
彼が嘘つき

682 :
結局、彼は嘘つきだった

A turned out to be X で
AはXであったのだ みたいな

683 :
すっきりしました、ありがとう御座いました。

684 :
The final match will be the fight between Rodriguez and Inoue. Donaire may be strong, but he does not pose as much of a threat as during his prime.

よろしくお願いします

685 :
決勝はロドリゲスと井上の戦いになるだろう。
ドネアも強いが、全盛期ほどの力は無い。

686 :
>>685
ありがとうございました

687 :
チャットで会話した相手の文章です

Yeah on another level of popular lol

翻訳にかけてもよくわかりませんでした

688 :
海外通販で初期不良の商品が届いたので、返金して貰ったのですが
返品手数料として$80.80引かれていたので、その$80.80は返金できないかと問い合わせた時の返答になります。
これは何を聞かれているのでしょうか?
So your refund was short by $80.80 due to the restocking fee charge right?

また、やり取りの中で、恐らく理由を説明してくれているのかなとは思うのですが
以下の回答がありました。
これはどういった内容なのかを教えて頂きたいです。
特に最後のwindowが何を指しているのかよく分かりません。
According to what I see the restocking fee was charged because the item was returned and received by us pass the return window.

よろしくお願いいたします。

689 :
>>687
うん、別のレベルの人気w
話の流れがわからんので細かい事はなんとも言えない

690 :
>>688
返品期限過ぎてたんじゃないの
return windowは返品期限
だからあんたの返金から手数料引かれてる事になっとんの、わかる?
うちが見たところ返品手数料引かれてるのは返品期限過ぎとるからやで
こんな感じでしょう

691 :
>>690
なるほど、返品期限の事だったんですね。
とても助かりました!
ありがとうございます。

692 :
windows ってそういう意味があったのか

>>688
返品手数料がかかるので返金が80ドル少なかった、ということですよね?

返品期間を過ぎての返品・到着のため返品手数料がかかっているようです。


翻訳以外は板違いだけど、返品に関する規約を参照した方がよさそう。返品期限が、こちらが連絡日なのか、発送した日なのか、相手が受け取った日なのか。
日本だと通常連絡日だと思う。

693 :
>>692
windowsじゃない、window。訂正。

694 :
>>689
ありがとうございました。分かました。
サッカーってアルゼンチンで人気だよね?という会話の流れからでした。

695 :
>>688
たとえば、$500の商品を買ったら不良品が届いた。返品したら手数料$80.80引かれて、$419.2しか戻ってこなかった。

$500戻ってくると期待したけど$80.80損した気分、という事だと思うけど、よく確認した方が良いと思う。おそらく、ちゃんと、文句言えば、$500丸々帰ってくるのと違うか?

696 :
Would you please stop to tell a lie, B?
I know exactly what's going on now and this is what I've DECIDED

697 :
嘘つくの辞めてもらっていいすかBさん
何が起こってるか理解してるし私が決めたことなんすよ
NARUTO?🤔

698 :
>>692
>>695
返品期限は確かに過ぎており、事前に返品が遅れそうな旨は連絡していたのですが、
その時は、「返品リクエストに対して注釈を付け加えておきます」みたいな回答をされたので、
勝手に大丈夫なのかな?と思っていたのですが、それが今回ダメだったようです...

何はともあれ、再度連絡してみたところ、無事に全額返金してもらう事が出来ました。
訳して下さった皆様、ありがとうございます!とても助かりました。

699 :
i see now , look i only play destiny and monster hunter . If it is something about any other game thats on my little brother

and you shoud teach him a lesson lol

So i can laugh at him hahah


ゲームにおいて、僕の友達が相手の友達に殺された事で、僕のチームが怒ってしまって、圧倒的に力量差があって、簡単に倒してしまう事になるので「あなたと戦いたくないから、物資を奪わないほうがいいよ」と忠告したらこれが来ました。

700 :
こんなクソガキのやり取り訳したくない
要はゲームのルール通り遊んどるんじゃボケナスって事や
挑発されてる攻撃してあげればいいんじゃないの

701 :
>>699
言ってることわかったよ。俺はDestinyとモンハンしかやらないんだ。
もし、俺の弟がやってるそれ以外のゲームに関することだったら、
遠慮なくお仕置きしてやってくれw
それを見て俺も笑うからw

702 :
For many of us, dreams are an almost intangible presence.
If we’re lucky, we can only remember the most fleeting glimpse in the cold light of day;
even those of us who can recollect past dreams in astonishing detail can wake some days
with almost no memory of what we had dreamed about.

703 :
I guess Alabama women can't be trusted to
deal with their pregnancies. Luckily for them,
they've got a bunch of men
(and one elderly lady) to tell them what to do.
#Handmaid'sTale.

海外ドラマ「ハンドメイズ・テイル/侍女の物語」
とアラバマの中絶禁止問題についてのツイートらしい
のですが翻訳お願いします

704 :
>>702
我々の多くにとって、夢というのは自体のない存在のようなものだ。
運がよければ、消え去る夢の一瞥を醒めている時間に思い出すことができる。
過去に見た夢を驚くべき正確さで思い出すことができる者も、日によっては、どんな夢を見たのかほとんど全く記憶がない状態で目覚めうる。

705 :
>>704
訂正
? 自体のない存在
○ 実体のない存在

706 :
>>704
ありがとう

707 :
Huawei will continue to provide security updates
to all existing Huawei smartphone
covering those have been sold or still in stock globally.

ここでのcoveringはsmartphoneを修飾していると解釈して大丈夫ですか?

708 :
Eren...tell me, how does it feel...
To have your own roof above your head again?!!

某人気マンガの英語版で、日本語でダジャレになってるセリフを
英語で別の意味のジョークにしているらしいんですがどういう意味でしょうか?
ちなみに元のセリフは「エレンの家ぇぇがあああ(エレン・イェーガーという名前と掛けている)」です。

709 :
もう少し付け足すと巨人に壊された複数の家が空から落ちてくるシーンです。
「空から家が・・・もしかしたらあの中にエレンの・・・エレンの家ぇぇがあああ」

ついでにこっちはアニメ版でのセリフです。
You think one of those houses... is Eren's?
Eren's house... is eavesdropping!

こっちはまた別のジョークになってるらしいですがこれもよくわかりません。
「落ちてくる」と盗聴をかけている?
だとしたらなぜ盗聴が出てきたのかもわかりません。

710 :
>>707
まず、質問とはそれるが、
ロイターの記事だと、covering those "that" have been ....のように関係代名詞thatがある。
こっちが正しい。テッククランチの記事だとなぜかthatが抜け落ちている。これはおかしい。

>ここでのcoveringはsmartphoneを修飾していると解釈して大丈夫ですか?

ちがう。これは分詞構文。coverinの主語にあたるのはHuawei

Huaweiはセキュリティアップデートを提供し続けるよ。すでに売れた端末にも、まだ売れてなくて
在庫としてある分も(Huaweiは)カバー(網羅)するよ。

711 :
>>708
まず、have a roof over one's head というイディオムがある。
直訳では「頭の上に屋根がある」つまりは「住む家がある」。
ギャンブルでほぼ全財産を失ってしまったが、家だけは売らずに済んだ、などという
場合、But at least I have a roof over my head. でも少なくともホームレスにならずに
、自分には住む家がある、などといえるだろう。

で、このイディオムと have your own roof above your headはaboveとover が異なる
だけで形は似ている。そこかダジャレになっている。頭の上から(above your head)
自分の家が降ってきているかもしれないので、over ではなく"above" your headになってる

英語圏の人から見ると、have your own roof above your head がhave your own roof over your head の変形
だとすぐわかり、ダジャレとして機能する。

712 :
>>709
Eren's house... is eavesdropping!

eaves には家のひさし、軒、という意味があるので、家が落ちてくる、家のひさしが落ちてくる、
という言葉遊びじゃないかな。

713 :
>>711-712
ありがとうございます!
これは調べてもわからない…
感謝します!

714 :
これお願いします
Most of からです


http://imgur.com/nUtOrMp.jpg

715 :
すみません、自己解決したので大丈夫です。

716 :
質問させてください。
日本人の妻がいて、日本に25年も住んでいるにも関わらず日本語が話せないPico Iyerさんについての記事です。
https://eikaiwa.dmm.com/app/daily-news/article/do-you-really-need-to-learn-japanese-to-live-in-japan/KpnSonXjEem3tL8-c3Z9SQ
Others were angry that the name of Iyer's "Japanese wife" was not given, even though she likely helps him survive in Japan.

ここの文の意味、特に「was not given」が分かりません。

717 :
その前のnameは「評判」って意味だと思う

718 :
It is the one who is called who comes.

719 :
>>716
妻の名前が書かれてない(=付属物扱いで個人扱いされてない)
どこに書かれてないのか書いてないのがこの文の不親切なところ

720 :
Gochu bro
Fr lol im trash rn
Ay hop on arcade foo

721 :
Operating under the theory that
the only stupid question is the one you don't ask
(a postulate with which some may disagree),
I pose this: Why is mayonnaise white?

翻訳お願いします

722 :
>>721
「愚かな質問があるとすればそれは実際に訊かれなかった質問だ」という考え方のもとに振る舞うとして
(これに同意しない人もいる仮定だとは思うが)
私はこう問う、「なぜマヨネーズは白いのか」と

723 :
1人の2ch運営の書き込みバイトスタッフがひたすら自作自演して
書き込んでるのが英語板
読むだけ無駄
あなたが読んでるレスの大半がこのバイトスタッフの書き込みだから
そうやって英語板に人がいるように見せてるだけ
でないと誰もいない廃墟の掲示板だとバレるから
書き込んだとしても
1.バイト
2.バイト
3.バイト
4.あなたの書き込み
5.バイト
6.バイト

あらゆるスレがこんな状態だから
大袈裟じゃなくてマジな話
1人のバイト工作員が人格変えながら複数の人を演じてるだけ

724 :
no Kim, he wasn't effin' with you.
He digs your music and probably was trying to get himself together and not go Groupie Fan on you.
You can tell he was a little struck as it looks like he gave you the card back without running it...


翻訳お願いします

725 :
>>722
ありがとうございます

726 :
英語板の工作員君のことがゲハで取り上げられてるよ
ついにゲハでもその自演の悪行が広まったか

アフィじゃなくて2ちゃんの運営が対立煽ってるだけというのが1番説得力あるよな
https://krsw.2ch.sc/test/read.cgi/ghard/1558698641/

727 :
高校生が学校で出題された和訳をしろ、というものですが…

Mr. Brown teaches mathematics and Mr. White English.

どう訳したら良いのでしょうか?
お願いします。

728 :
ミスターブラウンは数学を、ミスターホワイトは英語を教えている。

729 :
>>727
すみません、抜けてました。

Mr. Brown teaches us mathematics and Mr. White English.

>>728
ありがとうございます。
ですが、文法的に合っているのでしょうか?

730 :
これは本来は↓の英文の、繰り返しになる( )部分を省略したものだよ
Mr. Brown teaches us mathematics and Mr. White (teaches us) English.

731 :
>>730
自分は動作が無いじゃん、と思っていたのですが、
そこを省略可能なのですね。
理解しました。
ありがとうございました。

732 :
さらに以下の3つの英文の和訳をお願いします。

@The patients showed less depression and loneliness and lower levels of the strees hormone.

AIf you may not, lack of courage, you ask her to marry you, she will do some other man.

BShe always criticizes but never praises her children.

以上になります。よろしくお願いいたします。

733 :
>>732
@スペルミスです。
×strees
〇stress

734 :
1990'sから2000'sの芸術を英文にするとどうなりますか?

Fromの有無、toとuntilのニュアンスの違いについても伺いたいです

735 :
load-up top-up refill recharge reload チャージ(入金)
charge 電子マネー払い

初歩的なことらしいけど最近まで全然知らなかった
接客英語くらいはこなせると思ってただけにショックだ

736 :
誤爆しました

737 :
>>732
1:その患者達には抑鬱と孤独感の低減そしてストレスホルモンのレベル低下が見られた
2:勇気がなくて彼女にプロポーズしないなら彼女が他の男にプロポーズしちゃうよ
3:彼女はいつも粗探しばかりしてて1度たりとも自分の子供達を褒めたことがない
ask merry はいい日本語が思い浮かばないからプロポーズと多少意訳した
どれもなかなか酷いけど翻訳って結構難しいね

738 :
>>737
ありがとうございます。

1は、lessがlonelinessにも掛かっているんですね。
2は、do=askだったのですね。
3は、butが来ているけどもそういう訳になるのですね。む、難しいですね…汗

どれもこれも難しいものばかりで、勉強になりました。
ありがとうございました。

739 :
do = ask to merryと思ったけど文法に詳しい人がそのうち説明してくれると思う
言ってる事は「いつも避難する、"でも"褒めない」なので私の訳が意訳すぎたかも

740 :
>>739
恐れ入ります。

741 :
That Mr Trump ended up harming himself might be comforting for China.
That he went ahead despite knowing the risks should be less so.

お願いします

742 :
Extended metaphors are best avoided.
My advice is make a quick comparison
and then get on with it.

翻訳お願いします

743 :
政治の話は仕事でやってそうなので関わりたくないのでござる

744 :
>>717
>>719
ご回答ありがとうございます。理解のヒントにはなったのですが、やはりまだ完全に理解できず・・
別スレでも質問しみようとおもいます。ありがとうございました。

745 :2019/05/28
>>716
>>744
4月に、The New York Times が twitter で Pico Iyer の新著 Autumn Light (日本の歴史と文化についての本)へリンクし
「日本に長期間住む外国人は日本語を学ぶべきか」とツイートし、議論が巻き起こった。

そのツイートには何百ものレスがつき、多くの人が Iyer が25年間日本に住みながらほとんど日本語を話せないことに対して怒った。
彼が日本人と結婚していること(妻が彼を日本語面で助けていると思われる)が全く書かれていないことに対して怒った人たちもいた。


>>719が書いているように、どこに書かれていなかったのかが書かれていないので、複数の意味に取れてしまう悪文。
彼の新著に書かれていないのか、The New York Times または他人の tweet に書かれていないのか。

あたかも自分で日本の歴史や文化を理解した風を装いながら、実は妻にいろいろ助けてもらってる。
そのことが全く彼の本には書かれてない。という意味なのか、
外国人が外国で一人で survive するには、その国の言葉を学ぶ必要があるけれど、現地の妻が入れば話は別。
そんな重要なことが書かれていないなんて(彼の妻が日本人であることを考慮せずに、彼が日本語なしでやっていけてることを批判するなんて)。という意味なのか。

いずれにしても悪文なので深追いする価値はないかと。
追いたいならThe New York Timesのtweetとそれへのレスを読むほうがよくわかるかと。

外国人口説くレベルまで英会話力欲しい
おもてなし 即レス英会話
英検一級合格者のスレ
英語の発音総合スレ Part51
英会話教室・英会話学校 質問&雑談 THREAD 7
英語の勉強をやめることにしました。
英作の添削してちょ
セレンさんの経歴・英語力を検証するスレ12
悪徳業者NOVA(ノヴァ)を許さない
英検準1級スレ Part169
--------------------
『ろんろん亭♪』
【週刊現代】医療費タダ、受信料免除…生活保護は「年金暮らし」でも受け取れる ★2
オートバイ道って知ってますか
【朗報】野中ちぇるぴ痩せて可愛くなる
【自民党】『お肉券』、『お魚券』に批判殺到 農水相「国民の方々の反応、十分に受け止める」 → 林業などにも対象拡大へ ★3
韓国観光 男根公園
【コンピュータ将棋スレ】suimon応援スレ24【出禁】
【ホモ】銀魂なんでもあり雑談128【ノマ百合】
【ぐ】GUを自由に語ろう ジーユー82【g.u.】
【薪ストーブ】冬キャンプをしよう7【初心者歓迎】
フュージョンのまだCD化されていない名盤を挙げるスレ
【銚子市/加計】緊縮財政、60事業休廃止 銚子市予算案で5億2100万円削減
テレ朝チャンネル1 2 エンタメ〜テレ No.647
【圧巻】ニューヨークシティを真上から捉えた一枚が驚異的だと話題に [875850925]
フレンチポップス★2
Windows8 まとめスレ その1
リーマン板家族計画(*゚д゚)<ミルク代ホスィ
【芸能】野沢直子、老いを感じた出来事を明かす「いっぱい持って来ちゃう夜、、」
【リテラ】安倍首相が台風被害拡大の中「ラグビー」勝利に大はしゃぎツイート! 「夜を徹して救助」命じながら自分は私邸に帰り試合観戦
【愚痴】skyユーザーヲチスレ【Twitter】
TOP カテ一覧 スレ一覧 100〜終まで 2ch元 削除依頼